Tag Archives: 2020

Đề thi và đáp án kì thi chọn đội tuyển thi Quốc gia trường Phổ thông Năng khiếu năm học 2019 – 2020

ĐỀ THI

Ngày thi thứ nhất

Bài 1. Số thực $\alpha$ được gọi là điểm tụ của dãy số $\left(u_n\right)$ nếu tồn tại ít nhất một dãy con của $\left(u_n\right)$ có hội tụ đến $\alpha$.

(a) Hãy chỉ ra một dãy số có vô hạn điểm tụ.

(b) Chứng minh rằng nếu dãy số có mọi dãy con hội tụ thì nó cũng hội tụ.

(c) Gọi $S$ là tập hợp tất cả các số chính phương dương. Dãy số $\left(a_n\right)$ xác định bởi $a_n=\frac{1}{n}$ nếu $n \in S$ và $a_n=\frac{1}{n^2}$ nếu $n \notin S$.

Đặt $b_n=\sum_{k=1}^n a_k$. Xét tính hội tụ của các dãy số $\left(a_n\right)$ và $\left(b_n\right)$.

Bài 2. Tìm tất cả các hợp số dương $n$ sao cho $\sigma(n) \equiv 2(\bmod \varphi(n))$, trong đó ký hiệu $\sigma(n), \varphi(n)$ là hàm tổng các ước của $n$ và hàm Euler.

Bài 3. Tìm tất cả các hàm số $f: \mathbb{R} \rightarrow \mathbb{R}$ thỏa mãn

$\quad\quad\quad\quad\quad f(f(x)+y)+f(x) f(f(y))=x f(y)+x+y, \forall x, y \in \mathbb{R} .$

Bài 4. Cho tam giác $A B C$ không cân nội tiếp trong đường tròn $(O)$ với $B C$ cố định và $A$ thay đổi trên cung lớn $B C$. Các đường tròn bàng tiếp góc $A, B, C$ lần lượt tiếp xúc với $B C, C A, A B$ tại $D, E, F$. Gọi $L, M, N$ lần lượt là giao điểm khác $A, B, C$ của $(A B E),(A C F) ;(B C F),(B A D) ;(C A D),(C B E)$.

(a) Chứng minh rằng $A L$ luôn đi qua điểm cố định khi $A$ thay đổi.

(b) Gọi $K, I, J$ lần lượt là trung điểm của $A D, B E, C F$. Chứng minh rằng $K L, I M, J N$ dồng quy.

Ngày thi thứ hai

Bài 5. Cho $a, b, c$ là các số thực dương thỏa mãn $8\left(a^2+b^2+c^2\right)=9(a b+b c+c a)$.

Tìm giá trị lớn nhất và giá trị nhỏ nhất của biểu thức

$\quad\quad\quad\quad\quad\quad\quad\quad\quad\quad T=\frac{a+b}{c}+\frac{b+c}{a}+\frac{c+a}{b} .$

Bài 6. Tìm tất cả các hàm số $f: \mathbb{Z}^{+} \rightarrow \mathbb{Z}^{+}$thỏa mãn đồng thời các điều kiện sau

$\quad\quad$ i) $m f(m)+n f(n)+2 m f(n)$ là số chính phương với mọi $m, n$;

$\quad\quad$ ii) $f(m n)=f(m) f(n)$ với mọi $m, n$ nguyên dương;

$\quad\quad$  iii) Với mọi số nguyên tố $p, f(p)$ không chia hết cho $p^2$.

Bài 7. Một trường phổ thông có $n$ học sinh. Các học sinh tham gia vào tổng cộng $m$ câu lạc bộ là $A_1, A_2, \ldots, A_m$.

(a) Chứng minh rằng nếu mỗi câu lạc bộ có 4 học sinh và hai học sinh bất kỳ tham gia chung nhất một câu lạc bộ thì $m \leq \frac{n(n-1)}{12}$.

(b) Giả sử tồn tại $k>0$ sao cho hai câu lạc bộ bất kỳ có chung nhau $k$ thành viên và tồn tại một câu lạc bộ $A_t$ có $k$ thành viên. Chứng minh rằng $m \leq n$

Bài 8. Cho tam giác $A B C$ nội tiếp đường tròn $(O)$. Đường tròn nội tiếp $(I)$ tiếp xúc với các cạnh $B C, C A, A B$ lần lượt tại $D, E, F$. Gọi $J$ là tâm bàng tiếp góc $A$ của tam giác $A B C$ và $H$ là hình chiếu của $D$ lên $E F$.

(a) Chứng minh rằng giao điểm của $A H, J D$ thì thuộc đường thẳng $O I$.

(b) Giả sử $D H$ cắt lại $(I)$ ở $K$ và $I K$ cắt lại đường tròn ngoại tiếp $(I E F)$ ở $L$. Chứng minh rằng $A D, L H$ cắt nhau tại một điểm nằm trên $(I E F)$.

 

LỜI GIẢI

Ngày thi thứ nhất

Bài 1. Số thực $\alpha$ được gọi là điểm tụ của dãy số $\left(u_n\right)$ nếu tồn tại ít nhất một dãy con của $\left(u_n\right)$ có hội tụ đến $\alpha$.

(a) Hãy chỉ ra một dãy số có vô hạn điểm tụ.

(b) Chứng minh rằng nếu một dãy số có mọi dãy con hội tụ thì nó cũng hội tụ.

(c) Gọi $S$ là tập hợp tất cả các số chính phương dương. Dãy số $\left(a_n\right)$ xác định bởi $a_n=\frac{1}{n}$ nếu $n \in S$ và $a_n=\frac{1}{n^2}$ nếu $n \notin S$.

Đặt $b_n=\sum_{k=1}^n a_k$. Xét tính hội tụ của các dãy số $\left(a_n\right)$ và $\left(b_n\right)$.

Lời giải. (a) Ta sẽ chỉ ra dãy số mà mỗi số nguyên dương xuất hiện vô hạn lần trong đó. Chẳng hạn $\left(u_n\right)$ : là

$\quad\quad\quad\quad\quad\quad\quad\quad\quad\quad 1,2,1,2,3,1,2,3,4,1,2,3,4,5, \ldots$

với $u_n=1$ nếu $n \in S$ và $u_{n+1}=u_n+1$ nếu $n \notin S$, trong đó $S$ là tập hợp các số có dạng $\frac{m(m+1)}{2}$ như $1,3,6,10,15, \ldots$ Khi đó, với mỗi số nguyên dương $m \in \mathbb{Z}^{+}$thì ta luôn có thể trích ra một dãy con vô hạn của $\left(u_n\right)$ có tất cả các phần tử đều bằng $m$, tức là hội tụ về $m$.

(b) Do mỗi dãy số là dãy con của chính nó nên rõ ràng khẳng định của bài toán là đúng.

(c) Ta có $0 \leq a_n \leq \frac{1}{n}$ với mọi $n$ nên theo nguyên lí kẹp, ta suy ra $\lim a_n=0$. Nhận xét rằng $b_n$ là dãy tăng. Ta có

$\quad\quad\quad\quad\quad b_{n^2}=\sum_{i=1}^{n^2} a_i =\sum_{i \in S, i=1}^{n^2} a_i+\sum_{i \notin S, i=1}^{n^2} a_i=\sum_{i \in S, i=1}^{n^2} \frac{1}{i}+\sum_{i \notin S, i=1}^{n^2} \frac{1}{i^2} $

$\quad\quad\quad\quad\quad\quad\quad\quad\quad\quad\quad =\left(1+\frac{1}{2^2}+\cdots+\frac{1}{n^2}\right)+\sum_{i \notin S, i=1}^{n^2} \frac{1}{i^2} $

$\quad\quad\quad\quad\quad\quad\quad\quad\quad\quad\quad <\left(1+\frac{1}{2^2}+\cdots+\frac{1}{n^2}\right)+\left(\sum_{i=1}^{n^2} \frac{1}{i^2}\right) $

$\quad\quad\quad\quad\quad\quad\quad\quad\quad\quad\quad =\left(1+\frac{1}{2^2}+\cdots+\frac{1}{n^2}\right)+\left(1+\frac{1}{2^2}+\cdots+\frac{1}{n^4}\right)$

Vì dãy $u_n=1+\frac{1}{2^2}+\cdots+\frac{1}{n^2}<1+\frac{1}{1 \cdot 2}+\frac{1}{2 \cdot 3}+\cdots+\frac{1}{(n-1) n}=2-\frac{1}{n}<2$ là bị chặn trên nên từ đánh giá đã xây dựng được, ta có $b_{n^2}$ cũng bị chặn trên. Kết hợp với $b_n$ là dãy tăng, ta suy ra bản thân dãy $b_n$ cũng bị chặn trên nên nó hội tụ.

Bài 2. Tìm tất cả các hợp số dương $n$ sao cho

$\quad\quad\quad\quad\quad\quad\quad\quad\quad\quad \sigma(n) \equiv 2 \quad(\bmod \varphi(n)),$

trong đó ký hiệu $\sigma(n), \varphi(n)$ là hàm tổng các ước của $n$ và hàm Euler.

Lời giải . Giả sử $p$ là một ước nguyên tố lẻ của $n$. Nếu $v_p(n)>1$ thì theo công thức của hàm Euler, ta có $p \mid \varphi(n)$, mà $n \cdot \sigma(n)-2$ chia hết cho $\varphi(n)$, tức là cũng chia hết cho $p$ nên kéo theo $p \mid 2$, vô lý. Suy ra $v_p(n)=1$ với mọi $p \mid n$.

Đặt $n=2^k \cdot p_1 p_2 \ldots p_t$ với $k \geq 0$ và $p_1<p_2<\ldots<p_t$ là các số nguyên tố phân biệt. Theo công thức tính các hàm, ta có

$\quad\quad\quad\quad\quad\quad\quad \varphi(n)=2^{k-1}\left(p_1-1\right)\left(p_2-1\right) \ldots\left(p_t-1\right)$

$\quad\quad\quad\quad\quad\quad\sigma(n)=\left(2^{k+1}-1\right)\left(p_1+1\right)\left(p_2+1\right) \ldots\left(p_t+1\right) .$

Đánh giá lũy thừa 2 trong các số trên, ta có

$\quad\quad\quad\quad\quad\quad v_2(\varphi(n)) \geq k-1+t \text { và } v_2(n \cdot \sigma(n)) \geq k+t .$

Do đó từ $\varphi(n) \mid n \cdot \sigma(n)-2$, ta suy ra $1 \geq k-1+t$ nên $k+t \leq 2$. Ta xét các trường hợp sau

  • Nếu $t=0$ thì $n=2^k$ là hợp số nên $k=2, n=4$, thử trực tiếp ta thấy thỏa.
  • Nếu $t=1$ thì $n=2 p$ nên $\varphi(n)=p-1, \sigma(n)=3(p+1)$ và đưa về

$\quad\quad\quad\quad\quad\quad\quad\quad\quad\quad p-1 \mid 6 p(p+1)-2$

Chú ý rằng

$\quad\quad\quad\quad\quad\quad 6 p(p+1)-2=6 p^2+6 p-2=(p-1)(6 p+12)-10$

nên $p-1 \mid 10$. Từ đó ta tìm được $p=3, p=11$ tương ứng với $n=6, n=22$.

  • Nếu $t=2$ thì $k=0$, ta có $n=p_1 p_2$ nên

$\quad\quad\quad\quad \varphi(n)=\left(p_1-1\right)\left(p_2-1\right) \text { và } \sigma(n)=\left(p_1+1\right)\left(p_2+1\right)$

đưa về

$\quad\quad\quad\quad \left(p_1-1\right)\left(p_2-1\right) \mid\left(p_1+1\right)\left(p_2+1\right)-2 .$

Điều này không thể xảy ra vì $\left(p_1-1\right)\left(p_2-1\right)$ chia hết cho 4 trong khi biểu thức còn lại chia 4 dư 2 . Do đó, trường hợp này không có số $n$ thỏa mãn.

Vậy tất cả các số cần tìm là $4,6,22$.

Nhận xét. Chú ý rằng mọi số nguyên tố đều thỏa mãn yêu cầu của đề bài.

Bài 3. Tìm tất cả các hàm số $f: \mathbb{R} \rightarrow \mathbb{R}$ thỏa mãn

$\quad\quad\quad\quad\quad\quad f(f(x)+y)+f(x) f(f(y))=x f(y)+x+y$

với mọi số thực $x, y$.

Lời giải. Thay $x=y=0$ vào phương trình đề cho, ta có

$\quad\quad\quad\quad\quad\quad\quad\quad\quad\quad f(f(0))+f(0) f(f(0))=0 .$

suy ra $f(f(0))=0$ hoặc $f(0)=-1$. Ta xét các trường hợp sau:

  1. Nếu $f(f(0))=0$. Thay $y=0$, vào phương trình dề cho, ta có $f(f(x))=$ $x f(0)+x, \forall x \in \mathbb{R}$ Thay $x=f(0)$ và sử dụng $f(f(0))=0$, ta được $f(0)=$ $[f(0)]^2+f(0)$, hay $f(0)=0$. Do đó $f(f(x))=x$ với mọi $x \in \mathbb{R}$. Thay vào phương trình đề bài, ta có

$\quad\quad\quad\quad\quad\quad f(f(x)+y)+y f(x)=x f(y)+x+y, \forall x, y \in \mathbb{R} .$

Thay $y$ bởi $f(y)$ và sử dụng tính đối xứng của vế trái, ta được

$\quad\quad\quad\quad\quad\quad f(f(x)+f(y))+f(x) f(y)=x y+x+f(y)=x y+y+f(x) .$

Do đó $f(x)-x=f(y)-y$ với mọi $x, y \in \mathbb{R}$, hay $f(x)=x+c$. Thử lại, ta có $c=0$.

  1. Nếu $f(0)=-1$. Thay $y=0$ vào phương trình đề cho, ta có $f(f(x))+$ $f(x) f(-1)=0, \forall x \in \mathbb{R}$. Từ đây suy ra $f(f(-1))=-[f(-1)]^2$. Thay $x=0$ vào phương trình đề cho, ta có $f(y-1)-f(f(y))=y, \forall y \in \mathbb{R}$. Kết hợp các đẳng thức trên lại, ta có

$\quad\quad\quad\quad\quad\quad\quad\quad f(x-1)+f(x) f(-1)=x, \forall x \in \mathbb{R} .$

Thay $y=-1$ vào phương trình đề cho và sử dụng $f(f(-1))=-[f(-1)]^2$, ta lại có

$\quad\quad\quad\quad\quad\quad f(f(x)-1)-f(x)[f(-1)]^2=x f(-1)+x-1, \forall x \in \mathbb{R} .$

Mặt khác, ta cũng có

$\quad\quad\quad\quad\quad\quad\quad\quad f(-1) f(f(x))+f(x)[f(-1)]^2=0, \forall x \in \mathbb{R} .$

Cộng vế theo vế hai biểu thức trên lại, ta có

$\quad\quad\quad\quad\quad\quad\quad\quad\quad\quad f(x)=[1+f(-1)] x+1, \forall x \in \mathbb{R} .$

Thử lại, ta thấy không thỏa mãn.

Vậy phương trình có nghiệm hàm duy nhất là $f(x)=x$.

Bài 4. Cho tam giác $A B C$ không cân nội tiếp trong đường tròn $(O)$ với $B C$ cố định và $A$ thay đổi trên cung lớn $B C$. Các đường tròn bàng tiếp góc $A, B, C$ lần lượt tiếp xúc với các cạnh $B C, C A, A B$ tại $D, E, F$. Gọi $L, M, N$ lần lượt là giao điểm khác $A, B, C$ của các cặp đường tròn

$\quad\quad\quad\quad (A B E),(A C F) ;(B C F),(B A D) ;(C A D),(C B E) .$

(a) Chứng minh rằng $A L$ luôn đi qua điểm cố định khi $A$ thay đổi.

(b) Gọi $K, I, J$ lần lượt là trung điểm của $A D, B E, C F$. Chứng minh rằng $K L, I M, J N$ đồng quy.

Lời giải . (a) Đặt $B C=a, C A=b, A B=c$ và $p$ là nửa chu vi thì theo tính chất tiếp điểm bàng tiếp, ta có $B F=C E=p-a$.

Bằng biến đổi góc, ta có được $\triangle L B F \sim \triangle L E C(g . g)$, mà $B F=C E$ nên hai tam giác này bằng nhau. Suy ra $L B=L E, L C=L F$ nên $L$ là trung điểm cung $B E$ của đường tròn $(A B E)$ và cũng là trung diểm cung $C F$ của $(A C F)$.

Từ đó ta có $A L$ là phân giác góc $B A C$ hay $A L$ luôn đi qua trung điểm cung nhỏ $B C$ của $(O)$, là điểm cố định.

(b) Để ý rằng vai trò của $M, N, L$ là bình đẳng trong tam giác $A B C$. Do đó, từ câu a, một cách tương tự, ta có $M, N$ thuộc phân giác góc $B, C$ nên cũng lần lượt là trung điểm các cung nhỏ của các đường tròn tương ứng. Suy ra $M, K, N$ thẳng hàng (cùng thuộc trung trực của đoạn $A D$ ); tương tự với các bộ ba $N, I, L$ và $L, J, M$. Cuối cùng, ta thấy rằng

$\quad\quad\quad\quad\quad\quad\quad \frac{K M}{K N}=\frac{A K \cdot \tan \angle M A K}{A K \cdot \tan \angle N A K}=\frac{\tan (B / 2)}{\tan (C / 2)} .$

Tương tự với các tỷ số khác. Đến đây, áp dụng định lý Ceva cho tam giác $M N L$, ta có các đoạn thẳng $L K, I M, J N$ dồng quy.

Nhận xét. Một cách khác cho câu a như sau: Xét phép nghịch đảo đối xứng với phương tích $k=A B \cdot A C$ và trục đối xứng là phân giác góc $A$. Ta có $E \rightarrow E^{\prime} \in$ $A C, F \rightarrow F^{\prime} \in A B$ sao cho $A E \cdot A E^{\prime}=A F \cdot A F^{\prime}=k$. Ta tính được

$\quad\quad\quad\quad A E^{\prime}=\frac{b c}{p-c} \rightarrow B E^{\prime}=\frac{c(p-a)}{p-c} \rightarrow \frac{E^{\prime} B}{E^{\prime} A}=\frac{p-a}{b} .$

Tương tự thì $\frac{F^{\prime} C}{F^{\prime} A}=\frac{p-a}{c}$. Áp dụng định lý Ceva cho tam giác $A B C$ thì $C E^{\prime}, B F^{\prime}$ và phân giác góc $A$ đồng quy.

Lại có qua phép nghịch đối xứng trên thì phân giác giữ nguyên,

$\quad\quad\quad\quad\quad\quad\quad\quad\quad\quad (A B E) \rightarrow C F^{\prime},(A C F) \rightarrow B E^{\prime}$

nên ta có $L$ thuộc phân giác góc $A$.

 

Ngày thi thứ hai

Bài 5. Cho $a, b, c$ là các số thực dương thỏa mãn $8\left(a^2+b^2+c^2\right)=9(a b+b c+c a)$. Tìm giá trị lớn nhất và giá trị nhỏ nhất của biểu thức

$\quad\quad\quad\quad\quad\quad\quad\quad\quad\quad T=\frac{a+b}{c}+\frac{b+c}{a}+\frac{c+a}{b} .$

Lời giải . Do tính thuần nhất đối xứng của các biến nên chuẩn hóa

$\quad\quad\quad\quad\quad a b+b c+c a=8 \rightarrow a^2+b^2+c^2=9 \rightarrow a+b+c=5 .$

Ta có $P+3=(a+b+c)\left(\frac{1}{a}+\frac{1}{b}+\frac{1}{c}\right)=\frac{40}{a b c}$ nên ta đưa về tìm min, max của $T=a b c$ trong điều kiện

$\quad\quad\quad\quad\quad\quad\quad\quad\quad\quad\quad \left\{\begin{array}{l}a+b+c=5 \\ a b+b c+c a=8\end{array}\right.$

Chú ý rằng $b+c=5-a, b c=8-a(b+c)=8-a(5-a)$ nên từ đánh giá quen thuộc $(b+c)^2 \geq 4 b c$, ta có

$\quad\quad\quad\quad\quad\quad (5-a)^2 \geq 4\left(8-5 a+a^2\right) \Leftrightarrow 1 \leq a \leq \frac{7}{3} .$

Suy ra $T=a b c=a\left(8+a^2-5 a\right)=f(a)$. Đến đây khảo sát hàm số này trên miền $\left[1 ; \frac{7}{3}\right]$, ta được $\min T=4, \max T=\frac{112}{27}$ nên $\min P=\frac{93}{14}$, $\max P=7$. Từ đó, ta thu được kết luận như sau

  • Giá trị lớn nhất của $P$ là 7 , đạt được chẳng hạn khi $(a, b, c)=(2,2,1)$.
  • Giá trị nhỏ nhất của $P$ là $\frac{93}{14}$, đạt được chẳng hạn khi $(a, b, c)=\left(\frac{7}{3}, \frac{4}{3}, \frac{4}{3}\right)$.

Bài 6. Tìm tất cả các hàm số $f: \mathbb{Z}^{+} \rightarrow \mathbb{Z}^{+}$thỏa mãn đồng thời các điều kiện sau đây

$\quad\quad$ i) $m f(m)+n f(n)+2 m f(n)$ là số chính phương với mọi $m, n$;

$\quad\quad$  ii) $f(m n)=f(m) f(n)$ với mọi $m, n$ nguyên dương;

$\quad\quad$  iii) Với mọi số nguyên tố $p, f(p)$ không chia hết cho $p^2$.

Lời giải . Thay $m=n=1$ vào ii), ta suy ra $f(1)=f(1)^2$ nên $f(1)=1$. Thay $m=n$ vào i), ta suy ra $4 m f(m)$ là số chính phương với mọi $m \in \mathbb{Z}^{+}$nên $m f(m)$ cũng là số chính phương với mọi $m \in \mathbb{Z}^{+}$.

Với $p$ là số nguyên tố, vì $p f(p)$ là số chính phương nên $p \mid f(p)$ và ta đặt $f(p)=k^2 p$, với $k$ là số nguyên dương nào đó. Thay $m=p, n=1$ vào i), ta suy ra $p f(p)+1+2 p$ là số chính phương, hay $k^2 p^2+2 p+1$ là số chính phương.

Vì $k^2 p^2+2 p+1>(k p)^2$ nên

$\quad\quad\quad\quad\quad\quad k^2 p^2+2 p+1 \geq(k p+1)^2=k^2 p^2+2 k p+1 .$

Do đó $2 p \geq 2 k p$ nên ta phải có $k=1$.

Vì thế nên $f(p)=p$ với mọi số nguyên tố $p$. Sử dụng điều kiện ii), hàm $f$ nhân tính, và cũng vì mọi số nguyên dương bất kỳ đều có thể viết dưới dạng tích của các số nguyên tố nên ta có được $f(n)=n$ với mọi $n \in \mathbb{Z}^{+}$.

Thử lại ta thấy hàm số này thỏa mãn các ràng buộc của đề bài.

Nhận xét. Trên thực tế, ta có thể bỏ bớt diều kiện ii), iii) đi mà bài toán gốc vẫn có thể giải quyết được. Cụ thể như sau:

Chứng minh rằng nếu hàm số $f: \mathbb{Z}^{+} \rightarrow \mathbb{Z}^{+}$thỏa mãn $f(1)=1$ và với mọi $m, n \in \mathbb{Z}^{+}$, ta có $m f(m)+n f(n)+2 m f(n)$ là số chính phương thì $f(n)=n, \forall n \in \mathbb{Z}^{+}$.

Bài 7. Một trường phổ thông có $n$ học sinh. Các học sinh tham gia vào tổng cộng $m$ câu lạc bộ là $A_1, A_2, \ldots, A_m$.

(a) Chứng minh rằng nếu mỗi câu lạc bộ có 4 học sinh và hai học sinh bất kỳ tham gia chung nhất một câu lạc bộ thì $m \leq \frac{n(n-1)}{12}$.

(b) Giả sử tồn tại $k>0$ sao cho hai câu lạc bộ bất kỳ có chung nhau $k$ thành viên và tồn tại một câu lạc bộ $A_t$ có $k$ thành viên. Chứng minh rằng $m \leq n$

Lời giải . (a) Gọi $S$ là số bộ $({A, B}, C)$ mà trong đó học sinh $A, B$ cùng tham gia vào câu lạc bộ $C$. Ta thực đếm $S$ bằng hai cách

  1. Chọn câu lạc bộ trước, có $m$ cách, chọn cặp học sinh cùng tham gia vào đó có $C_4^2=6$ cách nên $S=6 \mathrm{~m}$.
  2. Chọn cặp học sinh trước, có $C_n^2$ cách, chọn câu lạc bộ mà hai học sinh đó cùng tham gia, có không quá 1 cách nên $S \leq C_n^2$.

Từ đó suy ra

$\quad\quad\quad\quad\quad\quad\quad\quad\quad 6 m \leq C_n^2 \Leftrightarrow m \leq \frac{n(n-1)}{12} .$

(b) Xét câu lạc bộ $X$ nào đó có $k$ thành viên. Xét $m-1$ câu lạc bộ còn lại thì theo giả thiết, rõ ràng các câu lạc bộ này đều có chứa $k$ thành viên trên của câu lạc bộ $X$. Từ đó suy ra $m-1$ câu lạc bộ còn lại đôi một không có thành viên chung.

Xét $n-k$ học sinh còn lại của trường thì rõ ràng một học sinh thuộc tối đa một câu lạc bộ (trong số các câu lạc bộ còn lại), suy ra số câu lạc bộ còn lại không vượt quá $n-k$ nên suy ra $m \leq n-k+1 \leq n$. Ta có điều phải chứng minh.

Nhận xét. Ý b của bài toán khá hiển nhiên, nhưng thực ra nó là một “phiên bản dễ” của bất đẳng thức Fisher sau đây:

Cho $A_1, A_2, \ldots, A_m$ là các tập con của tập ${1,2, \ldots, n}$ sao cho hai tập con bất kỳ có chung nhau đúng $k$ (với $k$ là số nguyên cố định nào đó không vượt quá n). Khi đó $m \leq n$.

Tuy nhiên, chứng minh sơ cấp cho kết quả này quả thực rất khó. Cách phổ biến nhất là dùng đại số tuyến tính. Cụ thể là:

Ta đặt tương ứng mỗi tập $A_i$ với một vector $v_i$ trong $\mathbb{F}_2^n$ như sau

$\quad\quad\quad\quad\quad\quad\quad\quad v_{i j}=\left\{\begin{array}{l}1 \text { nếu } j \in A_i \\ 0 \text { nếu } j \notin A_i\end{array} .\right.$

Chú ý rằng $\left|A_i \cap A_j\right|=k$ với mọi $i \neq j$. Bởi vậy, các vector $v_1, \ldots, v_m$ là các vector trong $\mathbb{R}^n$. Mặt khác, ta có số chiều của $\mathbb{R}^n$ là $n$. Do đó, trong bước tiếp theo chúng ta chỉ cần chứng minh $v_1, \ldots, v_m$ độc lập tuyến tính trong không gian $\mathbb{R}^n$.

Giả sử phản chứng rằng tồn tại các hệ số $\alpha_1, \ldots \alpha_m$ không đồng nhất bằng không sao cho $\sum_{i=1}^m \alpha_i v_i=0$. Do đó, ta có

$\quad\quad\quad\quad\quad\quad 0 =\left|\sum_{i=1}^m \alpha_i v_i\right|^2=\left\langle\sum_{i=1}^m \alpha_i v_i, \sum_{i=1}^m \alpha_i v_i\right\rangle $

$\quad\quad\quad\quad\quad\quad\quad =\sum_{i=1}^m \alpha_i^2\left|v_i\right|^2+\sum_{1 \leq i \neq j \leq m} \alpha_i \alpha_j\left\langle v_i, v_j\right\rangle$

Mặt khác, $\left|v_i\right|^2=\left|A_i\right|$, và $\left\langle v_i, v_j\right\rangle=\left|A_i \cap A_j\right|$. Bởi vậy,

$\quad\quad\quad\quad 0=\sum_{i=1}^m \alpha_i^2\left|v_i\right|^2+\sum_{i \neq j} k \alpha_i \alpha_j=\sum_{i=1}^m \alpha_i^2\left(\left|A_i\right|-k\right)+k \sum_{1 \leq i, j \leq m} \alpha_i \alpha_j$

Ta thấy rằng $\sum_{1 \leq i, j \leq m} \alpha_i \alpha_j=\left(\sum_{1 \leq i \leq m} \alpha_i\right)^2$, nên $0=\sum_{i=1}^m \alpha_i^2\left(\left|A_i\right|-k\right)+k\left(\sum_{1 \leq i, j \leq m} \alpha_i\right)^2$. Vì $\left|A_i\right| \geq k$ và có nhiều nhất một tập với chính xác $k$ phần tử, nên $\alpha_1=\cdots=\alpha_m=0$.

Điều này mâu thuẫn với giả thiết, hay các vector $v_1, \ldots, v_m$ là độc lập tuyến tính. Như vậy ta sẽ có $m \leq n$.

Bài 8. Cho tam giác $A B C$ nội tiếp đường tròn $(O)$. Đường tròn nội tiếp $(I)$ tiếp xúc với các cạnh $B C, C A, A B$ lần lượt tại $D, E, F$. Gọi $J$ là tâm bàng tiếp góc $A$ của tam giác $A B C$ và $H$ là hình chiếu của $D$ lên $E F$.

(a) Chứng minh rằng giao điểm của $A H, J D$ thì thuộc đường thẳng $O I$.

(b) Giả sử $D H$ cắt lại $(I)$ ở $K$ và $I K$ cắt lại đường tròn ngoại tiếp $(I E F)$ ơ $L$. Chứng minh rằng $A D, L H$ cắt nhau tại một diểm nằm trên $(I E F)$.

Lời giải. (a) Ta có bổ đề sau:

Bổ ĐỀ. $O I$ là đường thẳng Euler của tam giác $D E F$.

Bổ đề này quen thuộc và có thể chứng minh bằng cách hướng như sau (chi tiết xin dành cho bạn đọc).

  1. Sử dụng phép nghịch đảo tâm $I$, phương tích $r^2$ biến $(O)$ thành đường tròn Euler của $D E F$ nên có các tâm thẳng hàng.
  2. Sử dụng phép vị tự bằng cách gọi thêm trung điểm các cung nhỏ $B C, C A, A B$ của $(O)$.

Khi đó, gọi $T$ là giao điểm của $I O$ và $H D$ thì rõ ràng $T$ là trực tâm của tam giác $D E F$. Gọi $M$ là trung điểm cung nhỏ $B C$ của $(O)$ thì dễ thấy $M$ là trung điểm $I J$.

Bằng biến đổi góc, ta có $\triangle T E F \sim \triangle I B C$, mà $T H, I D$ là hai đường cao tương ứng nên $\frac{T H}{I D}=\frac{E F}{B C}$. Mặt khác, $\triangle I E F \sim \triangle M B C$ nên

$\quad\quad\quad\quad\quad\quad\quad\quad \frac{E F}{B C}=\frac{I E}{M C}=\frac{2 I E}{2 M I}=\frac{2 I E}{I J}$

suy ra $\frac{T H}{I D}=\frac{2 I E}{I J}$. Do đó

$\quad\quad\quad\quad\quad\quad T H \cdot I J=2 I D^2=2 I N \cdot I A=T D \cdot I A$

(vì $I, T$ lần lượt là tâm ngoại tiếp và trực tâm tam giác $D E F$ ) nên $\frac{T H}{T D}=\frac{I A}{I J}$. Cuối cùng, vì $H D | A J$ (cùng vuông góc với $E F$ ) nên theo định lý Talet thì $A H, J D, T I$ đồng quy hay nói cách khác, $A H, J D$ cắt nhau trên $O I$.

(b) Giả sử $A D$ cắt lại $(I)$ tại $G$. Ta cần chứng minh rằng $G, H, L$ thẳng hàng.

Do $D K | A I$ nên

$\quad\quad\quad\quad\quad\quad\quad\quad\quad\quad\angle A G L=\angle A I L=\angle A I K=\angle D K I$

suy ra $\angle D G L=\angle D K L$. Vì thế nên $D G K L$ là tứ giác nội tiếp. Do đó, $L G$ là trục đẳng phương của $(L K D),(I E F)$. Lại có

nên suy ra $H$ thuộc trục đẳng phương của hai đường tròn này, tức là $H \in L G$. Từ đó ta có điều phải chứng minh.

Nhận xét. Liên quan đến ý b, có một bài toán khá thú vị với nội dung như sau:

Trung tuyến đỉnh $D$ của tam giác $D E F$ cắt $(I)$ ở $L$. Chứng minh rằng trục đẳng phương của $(L B F),(L C E)$ đi qua giao điểm của $J D$ và đường thẳng qua $A$, vuông góc với $A I$.

 

 

 

 

 

 

 

 

 

 

 

 

 

 

 

 

 

 

 

 

 

 

 

 

 

 

 

 

 

 

 

 

 

 

 

 

 

 

 

 

 

 

 

 

 

 

 

 

 

 

 

 

 

 

 

 

 

 

 

 

 

 

 

 

 

 

 

 

 

 

 

 

 

 

 

 

 

 

 

 

 

 

 

 

 

 

 

 

 

 

 

 

 

 

 

 

 

 

 

 

Đề thi và đáp án kì thi chọn đội tuyển thi Quốc gia trường Phổ thông Năng khiếu năm học 2020 – 2021

ĐỀ THI

Ngày thi thứ nhất

Bài 1. Với mỗi số nguyên dương $n$, tìm số thực $M_n$ lớn nhất sao cho với mọi số thực dương $x_1, x_2, \ldots, x_n$ thì ta đều có

$\quad\quad\quad\quad\quad\quad\quad\quad\sum_{k=1}^n \frac{1}{x_k^2}+\frac{1}{\left(\sum_{k=1}^n x_k\right)^2} \geq M_n\left(\sum_{k=1}^n \frac{1}{x_k}+\frac{1}{\sum_{k=1}^n x_k}\right)^2$

Bài 2. Cho 2021 số nguyên khác 0 . Biết rằng tổng của một số bất kỳ trong chúng với tích của tất cả 2020 số còn lại luôn âm.

(a) Chứng minh rằng với mọi cách chia 2021 số này thành hai nhóm và nhân các số cùng nhóm lại với nhau thì tổng của hai tích cũng luôn âm.

(b) Một bộ số thỏa mãn đề bài thì có thể có nhiều nhất mấy số âm?

Bài 3. Cho hai hàm số $f: \mathbb{R} \rightarrow \mathbb{R}$ và $g: \mathbb{R} \rightarrow \mathbb{R}$ thỏa mãn $g(2020)>0$ và với mọi

$\quad\quad\quad\quad x, y \in \mathbb{R}$ thì $\left\{\begin{array}{l}f(x-g(y))=f(-x+2 g(y))+x g(y)-6 \\ g(y)=g(2 f(x)-y)\end{array}\right.$.

(a) Chứng minh rằng $g$ là hàm hằng.

(b) Chứng minh rằng đồ thị $h(x)=f(x)-x$ nhận $x=1$ là trục đối xứng.

Bài 4. Cho tam giác $A B C$ nhọn, nội tiếp trong đường tròn $(O)$ có trực tâm $H$ và $A H, B H, C H$ cắt cạnh đối diện lần lượt tại $D, E, F$. Gọi $I, M, N$ lần lượt là trung diểm các cạnh $B C, H B, H C$ và $B H, C H$ cắt lại $(O)$ theo thứ tự tại các điểm $L, K$. Giả sử $K L$ cắt $M N$ ở $G$.

(a) Trên $E F$, lấy diểm $T$ sao cho $A T$ vuông góc với $H I$. Chứng minh rằng $G T$ vuông góc với $O H$.

(b) Gọi $P, Q$ lần lượt là giao điểm của $D E, D F$ và $M N$. Gọi $S$ là giao điểm của $B Q, C P$. Chứng minh rằng $H S$ đi qua trung diểm của $E F$.

Ngày thi thứ hai

Bài 5. Cho số nguyên dương $n>1$. Chứng minh rằng với mọi số thực $a \in\left(0 ; \frac{1}{n}\right)$ và mọi đa thức $P(x)$ có bậc $2 n-1$ thỏa mãn điều kiện $P(0)=P(1)=0$, luôn tồn tại các số thực $x_1, x_2$ thuộc $[0 ; 1]$ sao cho $P\left(x_1\right)=P\left(x_2\right)$ và $x_2-x_1=a$.

Bài 6. Giải phương trình sau trên $\mathbb{Z}^{+}:\left(x^2+3\right)^{3^{x+1}}\left[\left(x^2+3\right)^{3^{x+1}}+1\right]+x^2+y=x^2 y$.

Bài 7. Cho các số nguyên $n>k>t>0$ và $X={1,2, \ldots, n}$. Gọi $\mathcal{F}$ là họ các tập con có $k$ phần tử của tập hợp $X$ sao cho với mọi $F, F^{\prime} \in \mathcal{F}$ thì $\left|F \cap F^{\prime}\right| \geq t$. Giả sử không có tập con có $t$ phần tử nào chứa trong tất cả các tập $F \in \mathcal{F}$.

(a) Chứng minh rằng tồn tại một tập hợp $B \subset X$ sao cho $|B|<3 k$ và $|B \cap F| \geq t+1$ với mọi $F \in \mathcal{F}$.

(b) Chứng minh rằng $|\mathcal{F}|<C_{3 k}^{t+1} C_n^{k-t-1}$.

Bài 8. Cho tam giác $A B C$ nội tiếp trong $(O)$ với $B, C$ cố định và $A$ thay đổi trên cung lớn $B C$. Dựng hình bình hành $A B D C$ và $A D$ cắt lại $(B C D)$ ở $K$.

(a) Gọi $R_1, R_2$ lần lượt là bán kính đường tròn ngoại tiếp $(K A B),(K A C)$. Chứng minh rằng tích $R_1 R_2$ không đổi.

(b) Ký hiệu $(T),\left(T^{\prime}\right)$ lần lượt là các đường tròn cùng đi qua $K$, tiếp xúc với $B D$ ở $B$ và tiếp xúc với $C D$ ở $C$. Giả sử $(T),\left(T^{\prime}\right)$ cắt nhau ở $L \neq K$. Chứng minh rằng $A L$ luôn đi qua một điểm cố định.

LỜI GIẢI

Ngày thi thứ nhất

Bài 1. Với mỗi số nguyên dương $n$, tìm số thực $M_n$ lớn nhất sao cho với mọi số thực dương $x_1, x_2, \ldots, x_n$ thì ta đều có

$\quad\quad\quad\quad\quad\quad\quad\quad\sum_{k=1}^n \frac{1}{x_k^2}+\frac{1}{\left(\sum_{k=1}^n x_k\right)^2} \geq M_n\left(\sum_{k=1}^n \frac{1}{x_k}+\frac{1}{\sum_{k=1}^n x_k}\right)^2 .$

Lời giải.

Điều Kiện Cần. Với $x_1=x_2=\cdots=x_n=1$, ta có

$\quad\quad\quad\quad\quad\quad\quad\quad n+\frac{1}{n^2} \geq M_n\left(n+\frac{1}{n}\right)^2 \text { hay } M_n \leq \frac{n^3+1}{\left(n^2+1\right)^2} .$

ĐIỀU KIỆN ĐỦ. Ta sẽ chứng minh hằng số trên thỏa mãn đề bài, tức là

$\quad\quad\quad\quad\quad\quad\quad\quad\sum_{k=1}^n \frac{1}{x_k^2}+\frac{1}{\left(\sum_{k=1}^n x_k\right)^2} \geq \frac{n^3+1}{\left(n^2+1\right)^2}\left(\sum_{k=1}^n \frac{1}{x_k}+\frac{1}{\sum_{k=1}^n x_k}\right)^2\quad\quad\quad\quad ( * )$

Theo bất đẳng thức Cauchy-Schwarz thì

$\quad\quad\quad\quad\quad\quad\quad\quad \sum_{k=1}^n \frac{1}{x_k^2} \geq \frac{1}{n}\left(\sum_{k=1}^n \frac{1}{x_k}\right)^2 .$

Đến đây, đặt $a=\sum_{k=1}^n \frac{1}{x_k}, b=\frac{1}{\sum_{k=1}^n x_k}$ là các số dương, ta đưa về chứng minh

$\quad\quad\quad\quad\quad\quad\quad\quad \frac{a^2}{n}+b^2 \geq \frac{n^3+1}{\left(n^2+1\right)^2}(a+b)^2 .$

Chú ý rằng $a \geq n^2 b$ nên quy đồng và khai triển, ta có

$\quad\quad\quad\quad\quad\quad\quad\quad \left(a-n^2 b\right)\left[\left(2 n^2-n+1\right) a-\left(n^3-n^2+2 n\right) b\right] \geq 0 .$

Chú ý rằng $n^2\left(2 n^2-n+1\right)-\left(n^3-n^2+2 n\right)=2\left(n^2+1\right) n(n-1) \geq 0$ nên

$\quad\quad\quad\quad\quad\quad\quad\quad \left(2 n^2-n+1\right) a \geq n^2\left(2 n^2-n+1\right) b \geq\left(n^3-n^2+2 n\right) b .$

Bất đẳng thức cuối đúng nên $(*)$ đúng. Vậy hằng số tốt nhất cần tìm là $M_{\max }=$ $\frac{n^3+1}{\left(n^2+1\right)^2}$

Nhận xét. Bài này là phiên bản tổng quát của câu bất đẳng thức trong đề Iran 2010 là

$\quad\quad\quad\quad\frac{1}{a^2}+\frac{1}{b^2}+\frac{1}{c^2}+\frac{1}{(a+b+c)^2} \geq \frac{7}{25}\left(\frac{1}{a}+\frac{1}{b}+\frac{1}{c}+\frac{1}{a+b+c}\right)^2 .$

Thực ra cách chứng minh cho trường hợp $n=3$ này cũng phản ánh cách xử lý cho trường hợp tổng quát trong bài toán ban đầu.

Bài 2. Cho 2021 số nguyên khác 0. Biết rằng tổng của một số bất kỳ trong chúng với tích của tất cả 2020 số còn lại luôn âm.

(a) Chứng minh rằng với mọi cách chia 2021 số này thành hai nhóm và nhân các số cùng nhóm lại với nhau thì tổng của hai tích cũng luôn âm.

(b) Một bộ số thỏa mãn đề bài thì có thể có nhiều nhất mấy số âm?

Lời giải: Đặt các số đã cho là $a_1, a_2, \ldots, a_{2021}$ và $S$ là tích của tất cả số này. Theo dề bài thì

$\quad\quad\quad\quad\quad\quad\quad\quad\quad\quad a_k+\frac{S}{a_k}=\frac{a_k^2+S}{a_k}<0$

với mọi $k=1,2, \ldots, 2021$.

Nếu như $S>0$ thì rõ ràng theo trên, ta phải có $a_k<0, \forall k$, nhưng điều này vô lý vì khi đó $S$ là tích của 2021 số âm nên cũng âm. Do đó, $S<0$ và trong các số đã cho, có lẻ số âm.

Nếu như số lượng số âm là lớn hơn 1 , giả sử hai trong các số đó là $a_1, a_2$ và $\left|a_2\right| \geq\left|a_1\right|$. Khi đó, ta có $a_2 a_3 \ldots a_{2021}>0$ (do trong các số này có chẵn số âm) và $\left|a_2 a_3 \ldots a_{2021}\right| \geq\left|a_1\right|$ nên

$\quad\quad\quad\quad\quad\quad\quad\quad\quad\quad a_1+a_2 a_3 \ldots a_{2021} \geq 0 .$

Điều vô lý này cho thấy không thể có nhiều hơn 1 số âm, và vì thế nên có đúng một số âm. Giả sử $a_1$ là số âm duy nhất đó thì ta có $a_1+a_2 a_3 \ldots a_{2021}<0$ nên

$\quad\quad\quad\quad\quad\quad\quad\quad\quad\quad \left|a_1\right|>a_2 a_3 \ldots a_{2021} .$

Với mọi cách chia 2021 số đã cho thành hai nhóm thì sẽ có một nhóm chứa số âm là $a_1$, đồng thời, giá trị tuyệt đối của tích các số trong nhóm đó, vì có chứa $\left|a_1\right|$, nên sẽ lớn hơn tích các số của nhóm còn lại. Suy ra tổng của hai tích sẽ âm.

Do đó, ta có khẳng định ở câu (a) và đáp số cho câu (b) là 1 .

Nhận xét. Nhờ việc khảo sát số lượng số âm, ta chỉ ra trực tiếp được rằng có đúng 1 số âm và các câu (a), (b) trở thành hiển nhiên. Bài toán cũng đúng khi thay 2021 số nguyên bởi $2 n+1$ số thực nào đó có giá trị tuyệt đối không nhỏ hơn 1 .

Bài 3. Cho hai hàm số $f: \mathbb{R} \rightarrow \mathbb{R}$ và $g: \mathbb{R} \rightarrow \mathbb{R}$ thỏa mãn $g(2020)>0$ và

$\quad\quad\quad\quad\quad\quad\quad\quad \left\{\begin{array}{l}f(x-g(y))=f(-x+2 g(y))+x g(y)-6 \\ g(y)=g(2 f(x)-y)\end{array}\right.$

với mọi $x, y \in \mathbb{R}$.

(a) Chứng minh rằng $g$ là hàm hằng.

(b) Chứng minh rằng đồ thị hàm số $h(x)=f(x)-x$ nhận $x=1$ là trục đối xứng.

Lời giải . (a) Trong điều kiện thứ nhất, thay $x$ bởi $\frac{3}{2} g(y)$ thì ta có ngay

$\quad\quad\quad\quad\quad\quad f\left(\frac{g(y)}{2}\right)=f\left(\frac{g(y)}{2}\right)+\frac{3}{2} g^2(y)-6 \text { hay } g^2(y)=4$

Suy ra $g(y)=\pm 2$ với mọi $y$. Do $g(2020)>0$ nên loại trường hợp $g(y)=-2, \forall y$. Ta sẽ chỉ ra rằng không xảy ra trường hợp $\exists a \neq b$ sao cho $g(a)=2, g(b)=-2$. Thay $y=2020$ vào điều kiện đầu, ta có

$\quad\quad\quad\quad\quad\quad\quad\quad\quad\quad f(x-c)-f(-x+2 c)=x c-6$

với $c=g(2020)>0$. Suy ra $f(u)-f(v)$ toàn ánh trên $\mathbb{R}$ với $u, v \in \mathbb{R}$. Tiếp theo trong diều kiện sau, thay $y$ bởi $a, b$, ta có

$\quad\quad\quad\quad\quad\quad g(a)=g(2 f(x)-a)=2 ; g(b)=g(2 f(x)-b)=-2$

với mọi $x \in \mathbb{R}$. Lại do tính toàn ánh, chọn $u, v$ sao cho

$\quad\quad\quad\quad\quad\quad\quad\quad\quad\quad f(u)-f(v)=\frac{a-b}{2}$

thì $2 f(u)-a=2 f(v)-b$. Suy ra

$\quad\quad\quad\quad\quad\quad\quad 2=g(2 f(u)-a)=g(2 f(v)-b)=-2,$

điều này vô lý. Vì thế $g(x)=2, \forall x$ nên $g$ là hàm hằng.

(b) Với $g(x)=2, \forall x$, thay vào điều kiện đầu, ta có

$\quad\quad\quad\quad\quad\quad\quad\quad f(x-2)=f(-x+4)+2 x-6$

hay

$\quad\quad\quad\quad f(x)=f(2-x)+2 x-2 \text { kéo theo } f(x)-x=f(2-x)-(2-x) .$

Điều này cho thấy $h(x)=h(2-x)$ hay $h(x)$ có đồ thị nhận $x=1$ là trục đối xứng.

Bài 4. Cho tam giác $A B C$ nhọn, nội tiếp trong đường tròn $(O)$ có trực tâm $H$ và $\mathrm{AH}, \mathrm{BH}, \mathrm{CH}$ cắt cạnh đối diện lần lượt tại $D, E, F$. Gọi $I, M, N$ lần lượt là trung điểm các cạnh $B C, H B, H C$ và $B H, C H$ cắt lại $(O)$ theo thứ tự tại các điểm $L, K$. Giả sử $K L$ cắt $M N$ ở $G$.

(a) Trên $E F$, lấy điểm $T$ sao cho $A T$ vuông góc với $H I$. Chứng minh rằng $G T$ vuông góc với $O H$.

(b) Gọi $P, Q$ lần lượt là giao điểm của $D E, D F$ và $M N$. Gọi $S$ là giao điểm của $B Q, C P$. Chứng minh rằng $H S$ di qua trung điểm của $E F$.

Lời giải. (a) Giả sử tia $I H$ cắt $(O)$ ở $R$ thì theo kết quả quen thuộc, ta có $\angle A R H=$ $90^{\circ}$. Vì thế nên $T \in A R$. Bằng cách xét trục đẳng phương của các đường tròn đường kính $A H, B C$ và đường tròn $(O)$, ta có $A R, E F, B C$ dồng quy. Từ đó suy ra $T \in B C$.

Gọi $\left(O^{\prime}\right)$ là đường tròn Euler của tam giác $A B C$ thì $D, E, F, M, N \in(O)$. Dễ thấy rằng

$\quad\quad\quad\quad H M \cdot H K=\frac{1}{2} H B \cdot H K=\frac{1}{2} H C \cdot H L=H N \cdot H L$

nên $M, N, K, L$ cùng thuộc đường tròn. Suy ra

$G L \cdot G K=G M \cdot G N$ nên

Ngoài ra, ta cũng có nên $G T$ chính là trục đăng phương của $(O),\left(O^{\prime}\right)$. Điều này cho thấy $G T \perp O O^{\prime}$ hay $G T \perp O H$ (do $O^{\prime}$ là trung diểm của $\left.O H\right)$.

(b) Ta có $D H$ là phân giác của góc $P D Q$, và $P Q \perp H D$ nên dễ thấy tứ giác $H P D Q$ là hình thoi. Ta biến đổi góc như sau

$\quad\quad\quad\quad\quad\quad\quad\quad \angle H P Q=\angle D Q P=\angle Q D B=\angle F H B .$

Suy ra $\angle H P N=\angle M H N$ nên $H N$ tiếp xúc với $(H M P)$ hay $N P \cdot N M=N H^2=$ $N C^2$. Do đó, hai tam giác $N P C$ và $N C M$ dồng dạng với nhau. Suy ra

$\quad\quad\quad\quad\quad\quad\quad\quad\quad\quad \angle N C P=\angle N M C=\angle M C B,$

nên $C P$ là dối trung của tam giác $H B C$. Chứng minh tương tự thì $B Q$ là dối trung trong tam giác $H B C$ nên điểm $S$ chính là điểm Lemoine của tam giác này, kéo theo $H S$ cũng là dối trung của tam giác $H B C$. Lại có $E F$ là dối song ứng với đỉnh $H$ trong tam giác $H B C$ nên suy ra $H S$ chia đôi đoạn thẳng $E F$.

 

Ngày thi thứ hai

Bài 5. Cho số nguyên dương $n>1$. Chứng minh rằng với mọi số thực $a \in$ $\left(0 ; \frac{1}{n}\right)$ và mọi đa thức $P(x)$ có bậc $2 n-1$ thỏa mãn điều kiện $P(0)=P(1)=0$, luôn tồn tại các số thực $x_1, x_2$ thuộc $[0 ; 1]$ sao cho $P\left(x_1\right)=P\left(x_2\right)$ và $x_2-x_1=a$.

Lời giải. Không mất tính tổng quát, giả sử không tồn tại các số $x_1, x_2$ thỏa mãn đề bài. Khi đó, xét đa thức $Q(x)=P(x+a)-P(x)$ sẽ vô nghiệm trên $[0 ; 1-a]$. Suy ra $Q(x)$ sẽ không đổi dấu trên miền đó vì nếu không, dùng tính liên tục thì sẽ mâu thuẫn. Không mất tính tổng quát, giả sử $Q(x)>0, \forall x \in[0 ; 1-a]$. Nhận xét rằng $\frac{1}{a} \notin \mathbb{Z}^{+}$vì nếu không, đặt $\frac{1}{a}=m \in \mathbb{Z}^{+}$thì ta sẽ có $m a=1$ và

$\quad\quad\quad\quad Q(a)+Q(2 a)+\cdots+Q((m-1) a)=P(1)-P(0)=0$

trong khi các số hạng ở vế trái đều dương, vô lý. Tiếp theo, ta có

$\quad\quad\quad\quad\quad\quad Q(a)=P(a)-P(0)>0 \Rightarrow P(a)>0$

Chứng minh tương tự thì $P(k a)>0, \forall k=1,2, \ldots, n$. Mặt khác,

$\quad\quad\quad\quad Q(1-a)=P(1)-P(1-a)=-P(1-a)>0 \text { nên } P(1-a)<0 .$

Tương tự thì

$\quad\quad\quad\quad\quad\quad\quad\quad\quad\quad P(1-l a)<0, \forall l=1,2, \ldots, n .$

Rõ ràng với mỗi $k \in{1,2, \ldots, n}$, ta luôn chọn được số nguyên $l_k \in{1,2, \ldots, n}$ để $(k-1) a<1-l_k a<k a$. Thật vậy, đánh giá này tương đương với

$\quad\quad\quad\quad\quad\quad\quad\quad\quad\quad \frac{1}{a}-k<l_k<\frac{1}{a}+1-k .$

Mà $a \in\left(0 ; \frac{1}{n}\right)$ nên $\frac{1}{a}-k>0$ và $\frac{1}{a}-k \notin \mathbb{Z}$ nên khoảng trên phải chứa một số nguyên và gọi số đó là $l_k$. Theo trên thì

$\quad\quad\quad\quad\quad\quad P((k-1) a)>0, P\left(1-l_k a\right)<0, P(k a)>0$

nên trên khoảng $((k-1) a ; k a)$ thì đa thức $P(x)$ có 2 nghiệm thực phân biệt. Áp dụng điều này cho tất cả các khoảng $(0 ; a),(a ; 2 a), \ldots,((n-1) a ; n a)$ thì ta thấy $P(x)$ sẽ có $2 n$ nghiệm thực phân biệt; trong khi $P(x)$ chỉ có bậc $2 n-1$, vô lý.

Vì thế nên điều giả sử là sai và tồn tại hai số thực $x_1, x_2 \in[0 ; 1]$ thỏa mãn đề bài.

Nhận xét. Ta thấy rằng do $P(x)$ đã có sã̃n hai nghiệm là $x=0, x=1$ nên lập luận trên vẫn đúng khi xét bậc đa thức là $2 n+1$ (vì ta đã chỉ ra được đến $2 n+2$ nghiệm phân biệt). Một phiên bản tương tự khi xét hàm số liên tục $f(x)$ tùy ý là:

Cho số nguyên dương $n$, xét hàm số $f(x)$ liên tục tục trên $[0 ; n]$ sao cho $f(0)=f(n)$. Khi đó, với $k \in{1,2, \ldots, n-1}$, tồn tại $x_1, x_2 \in[0 ; n]$ mà

$\quad\quad\quad\quad\quad\quad\quad\quad\quad\quad x_2-x_1=k \text { và } f\left(x_1\right)=f\left(x_2\right) .$

Bài 6. Giải phương trình sau trên tập số nguyên dương

$\quad\quad\quad\quad\quad\left(x^2+3\right)^{3^{x+1}}\left[\left(x^2+3\right)^{3^{x+1}}+1\right]+x^2+y=x^2 y .$

Lời giải . Dễ thấy $x=1$ không thỏa nên ta xét $x>1$. Thử trực tiếp thấy $x=2$ thỏa mãn. Xét $x>2$, kéo theo $x^2-1>3$. Phương trình đã cho viết lại thành

$\quad\quad\quad\quad\left(x^2+3\right)^{3^{x+1}}\left[\left(x^2+3\right)^{3^{x+1}}+1\right]+1=(y-1)\left(x^2-1\right) $

$\quad\quad\quad\quad \Leftrightarrow x^2-1\mid\left(x^2+3\right)^{3^{x+1}}\left[\left(x^2+3\right)^{3^{x+1}}+1\right]+1 $

$\quad\quad\quad\quad \Leftrightarrow 4^{3^{x+1}}\left(4^{3^{x+1}}+1\right)+1 \equiv 0 \quad\left(\bmod x^2-1\right) .$

Đặt $a=4^{3^{x+1}}$ thì ta có $a^2+a+1 \equiv 0\left(\bmod x^2-1\right)$ nên $a^3 \equiv 1\left(\bmod x^2-1\right)$ hay

$\quad\quad\quad\quad\quad\quad\quad\quad\quad\quad 4^{3^{x+2}} \equiv 1 \quad\left(\bmod x^2-1\right)$

Lại chú ý rằng
$\quad\quad\quad\quad\quad\quad a=4^{3^{x+1}}=4^{3 \cdot 3^x}=\left(4^3\right)^{3^x} \equiv 1 \quad(\bmod 9)$
nên nếu đặt $M=a^2+a+1$ thì $M \equiv 3(\bmod 9)$. Điều này cho thấy $v_3(M)=1$ nên $v_3\left(x^2-1\right) \leq 1$. Mà $x^2-1>3$ và dễ thấy $x^2-1$ lẻ nên $x^2-1$ phải có một ước nguyên tố lẻ $p>3$.

Đặt $h=\operatorname{ord}_p(4)$ thì $h \mid p-1$ và $h \mid 3^{x+2}$. Suy ra $h=3^k$ với $0 \leq k \leq x+2$. Tuy nhiên, nếu $k \leq x+1$ thì $4^{3^k} \equiv 1(\bmod p)$ mà $4^{3^k}-1 \mid a-1$ nên $a \equiv 1(\bmod p)$. Điều này vô lý do sẽ kéo theo
$\quad\quad\quad\quad\quad\quad\quad\quad\quad\quad 0 \equiv a^2+a+1 \equiv 3 \quad(\bmod p) .$
Vì thế nên $k=x+2$ hay $\operatorname{ord}_p(4)=3^{x+2}$ nên $h=3^{x+2} \leq p-1 \leq x-1$. Tuy nhiên, đánh giá này là không thể xảy ra với mọi $x>2$. Vì thế nên nghiệm duy nhất của phương trình dã cho là

$\quad\quad\quad\quad\quad\quad\quad\quad (x ; y)=\left(2 ; \frac{7^{27}\left(7^{27}+1\right)+4}{3}\right)$.

Nhận xét. Các bài toán lũy thừa tầng trong số học thường gợi ý đến việc dùng cấp theo kiểu: với mọi $a, m, k$ nguyên dương lớn hơn 1 và $p$ là số nguyên tố thì $a^{m^k} \equiv 1$ $(\bmod p)$ và $h=\operatorname{ord}_p(a)$ thì $h \mid m^k$ nên có $h=m^t$ với $t \leq k$. Một kết quả có liên quan là: mọi ước nguyên tố $p$ của $2^{2^n}+1$ thì đều thỏa $2^{n+1} \mid p-1$.

Bài 7. Cho các số nguyên $n>k>t>0$ và $X={1,2, \ldots, n}$. Gọi $\mathcal{F}$ là họ các tập con có $k$ phần tử của tập hợp $X$ sao cho với mọi $F, F^{\prime} \in \mathcal{F}$ thì $\left|F \cap F^{\prime}\right| \geq t$. Giả sử không có tập con có $t$ phần tử nào được chứa trong tất cả các tập $F \in \mathcal{F}$.

(a) Chứng minh rằng tồn tại một tập hợp $B \subset X$ sao cho $|B|<3 k$ và $|B \cap F| \geq t+1$ với mọi $F \in \mathcal{F}$.

(b) Chứng minh rằng $|\mathcal{F}|<C_{3 k}^{t+1} C_n^{k-t-1}$.

Lời giải. (a) Theo giả thiết thì rõ ràng $|\mathcal{F}| \geq 3$, vì nếu không thì diều kiện ii) sẽ không được thỏa mãn. Ta xét các trường hợp sau Nếu như mọi tập $F, F^{\prime} \in \mathcal{F}$ đều có tính chất $\left|F \cap F^{\prime}\right| \geq t+1$ thì ta chỉ cần chọn $B$ là một tập bất kỳ trong $\mathcal{F}$ là được, rõ ràng $|B|=k<3 k$, thỏa mãn đề bài.

Ngược lại, tồn tại hai tập $F, F^{\prime} \in \mathcal{F}$ mà $\left|F \cap F^{\prime}\right|=t$ thì theo giả thiết, không có tập con $t$ phần tử nào được chứa trong tất cả tập hợp của họ $\mathcal{F}$; vì thế nên phải có $F^{\prime \prime}$ sao cho $\left(F \cap F^{\prime}\right) \not \subset F^{\prime \prime}$. Mặt khác, vì $\left|F^{\prime \prime}\right|=k>t$ nên phải có phần tử trong $F^{\prime \prime}$ mà không thuộc vào $F \cap F^{\prime}$. Khi đó, xét $B=F \cup F^{\prime} \cup F^{\prime \prime}$ thì rõ ràng

$\quad\quad\quad\quad\quad |B| \leq\left|F \cup F^{\prime}\right|+\left|F^{\prime \prime}\right| \leq 2 k-t+k<3 k .$

Ta sẽ chứng minh rằng tập hợp này thỏa mãn đề bài. Thật vậy, xét tập hợp $G \in \mathcal{F}$

  • Nếu $G$ là một trong ba tập $F, F^{\prime}, F^{\prime \prime}$ thì có ngay

$\quad\quad\quad\quad\quad\quad\quad\quad\quad\quad |B \cap G| \geq k \geq t+1$

  • Nếu $G \neq F, F^{\prime}, F^{\prime \prime}$ thì nếu $|G \cap F| \geq t+1$ hoặc $\left|G \cap F^{\prime}\right| \geq t+1$ là xong; ngược lại $|G \cap F|=t$ và $t$ phần tử chung này không đồng thời thuộc $F^{\prime \prime}$, nên sẽ còn một phần tử chung khác giữa hai tập hợp $G, F^{\prime^{\prime}}$ do ta có $\left|G \cap F^{\prime \prime}\right| \geq t$, kéo theo $|B \cap G| \geq t+1$.

Trong mọi trường hợp, ta luôn chọn được tập hợp $B \subset X$ thỏa mãn đề bài.

(b) Ta thực hiện xây dựng một họ $\mathcal{F}^{\prime}$ các tập con của $X$ như sau

  • Chọn $t+1$ phần tử nào đó trong tập $B$ ở trên, số cách chọn sẽ nhỏ hơn $C_{3 k}^{t+1}$.
  • Chọn $k-t-1$ phần tử nào đó trong tập $X$ thì có $C_n^{k-t-1}$ cách.

Hợp của hai nhóm này lại thì được một tập con của $X$ có $k$ phần tử (hoặc ít hơn) mà giao với $B$ là $t+1$ phần tử. Từ đó dễ thấy rằng $\mathcal{F} \subset \mathcal{F}^{\prime}$ nên ta phải có $|\mathcal{F}| \leq\left|\mathcal{F}^{\prime}\right|<C_{3 k}^{t+1} C_n^{k-t-1}$. Bài toán được giải quyết.

Nhận xét. Cố định $k, t \geq 2$ và cho $n$ đủ lớn thì ta có

$\quad\quad\quad\quad\quad\quad\quad\quad\quad\quad C_{3 k}^{t+1} C_n^{k-t-1}<C_{n-t}^{k-t} .$

Từ đó, ta có chứng minh của định lý Erdós – Ko – Rado (1938) là: $|\mathcal{F}| \leq C_{n-t}^{k-t}$ với $n$ đủ lớn. Trường hợp $t=1$ thì khi $n \geq 2 k$, ta cũng có $|\mathcal{F}| \leq C_{n-1}^{k-1}$.

Các bài toán theo kiểu family set đã được nghiên cứu rất sâu sắc và vì hầu hết là các đánh giá, ước lượng tổ hợp không có dấu bằng nên đó đều là các kết quả khó.

Bài 8. Cho tam giác $A B C$ nội tiếp trong đường tròn $(O)$ với $B, C$ cố định và $A$ thay đổi trên cung lớn $B C$. Dựng hình bình hành $A B D C$ và $A D$ cắt lại $(B C D)$ ở $K$.

(a) Gọi $R_1, R_2$ lần lượt là bán kính đường tròn ngoại tiếp $(K A B),(K A C)$. Chứng minh rằng tích $R_1 R_2$ không đổi.

(b) Ký hiệu $(T),\left(T^{\prime}\right)$ lần lượt là các đường tròn cùng đi qua $K$, tiếp xúc với $B D$ ở $B$ và tiếp xúc với $C D$ ở $C$. Giả sử $(T),\left(T^{\prime}\right)$ cắt nhau ở $L \neq K$. Chứng minh rằng $A L$ luôn đi qua một điểm cố định.

Lời giải. (a) Gọi $R$ là bán kính của $(O)$, ta có

$\quad\quad\quad\quad\quad\quad\quad\quad \angle A K B=180^{\circ}-\angle B K D=\angle B C D=\angle A B C$

nên theo định lý sin thì

$\quad\quad\quad\quad\quad\quad\quad\quad R_1=\frac{A B}{2 \sin A K B}=\frac{2 R \sin C}{2 \sin B}=R \cdot \frac{\sin C}{\sin B} .$

Tương tự thì $R_2=R \cdot \frac{\sin B}{\sin C}$ nên $R_1 R_2=R^2$, không đổi.

(b) Do các tiếp tuyến nên biến đổi góc được

$\quad\quad\quad\quad\quad\quad \angle K L B=180^{\circ}-\angle K B D, \angle K L C=180^{\circ}-\angle K C D .$

Suy ra

$\quad\quad\quad\quad\quad\quad \angle K L B+\angle K L C=360^{\circ}-(\angle K B D+\angle K C D)=180^{\circ}$

nên $B, L, C$ thẳng hàng. Do $B D$ tiếp xúc với $(K B L)$ nên

$\quad\quad\quad\quad\quad\quad \angle B K L=\angle L B D=\angle C K D,$

mà $K D$ là trung tuyến của tam giác $K B C$ nên $K L$ là đối trung của tam giác này. Gọi $I$ là trung điểm $B C$. Ta có

$\quad\quad\quad\quad\quad\quad \angle K B I=\angle K D C=\angle B A K$

nên $B C$ tiếp xúc với $(A B K)$. Suy ra $\frac{B K}{A B}=\frac{I K}{I A}$. Tương tự thì

$\quad\quad\quad\quad\quad\quad \frac{C K}{A C}=\frac{I K}{I A} \text { nên } \frac{K B}{K C}=\frac{A B}{A C} .$

Theo tính chất quen thuộc của đường đối trung thì

$\quad\quad\quad\quad\quad\quad \frac{L B}{L C}=\frac{K B^2}{K C^2}=\frac{A B^2}{A C^2}$

nên $A L$ cũng là đường đối trung của tam giác $A B C$, diều này cho thấy $A L$ đi qua giao điểm hai tiếp tuyến của $(O)$ ở $B, C$, là điểm cố định.

Nhận xét. Điểm $K$ trong đề bài chính là điểm Humpty của tam giác $A B C$, và cách dựng như trên là một cách tuy dễ nhưng tương đối mới.

 

 

 

 

 

 

 

 

 

 

 

 

 

 

 

 

 

 

 

 

 

 

 

 

 

 

 

 

 

 

 

 

 

 

 

 

 

 

 

 

 

 

 

 

 

 

 

 

 

 

 

 

 

 

 

 

 

 

 

 

 

 

 

 

 

 

 

 

 

 

 

 

 

 

 

 

 

 

 

 

 

 

 

 

 

 

 

 

 

 

 

ĐỀ THI VÀO LỚP 10 CHUYÊN TOÁN TP.HCM NĂM 2020

Bài 1. Cho ba số dương $a, b, c$ thỏa mãn điều kiện $\frac{a}{b+c}+\frac{b}{c+a}+\frac{c}{a+b}=2020$ Tính giá trị của biểu thức $P=\left(\frac{a^{2}}{b+c}+\frac{b^{2}}{c+a}+\frac{c^{2}}{a+b}\right):(a+b+c)$

Bài 2. (a) Giải phương trình: $\sqrt{2 x^{2}+x+9}+\sqrt{2 x^{2}-x+1}=x+4$

(b) Giải hệ phương trình: $\left\{\begin{array}{l}y^{2}-2 x y=8 x^{2}-6 x+1 \\ y^{2}=x^{3}+8 x^{2}-x+1\end{array}\right.$

Bài 3. Cho tam giác nhọn $A B C(A B<B C<C A)$ nội tiếp đường tròn $(O)$. Từ $A$ kẻ đường thẳng song song với $B C$ cắt $(O)$ tại $A_{1}$. Từ $B$ kẻ đường thẳng song song với $A C$ cắt $(O)$ tại $B_{1}$. Từ $C$ kẻ đường thẳng song song với $A B$ cắt $(O)$ tại $C_{1}$. Chứng minh rằng các đường thẳng qua $A_{1}, B_{1}, C_{1}$ lần lượt vuông góc với $B C$, $C A, A B$ đồng quy.

Bài 4. (a) Cho 2 số thực $a, b$. Chứng minh rằng: $\frac{a^{2}+b^{2}}{2} \geq a b+\frac{(a-b)^{2}}{a^{2}+b^{2}+2}$

(b) Cho hai số dương $a, b$ thỏa mãn điều kiện $a+b \leq 3$

Tìm giá trị nhỏ nhất của biểu thức: $Q=b-a+\frac{20}{a}+\frac{7}{b}$.

Bài 5. Đường tròn $(I)$ nội tiếp tam giác $A B C$ tiếp xúc với các cạnh $A B, B C, C A$ lần lượt tại $D, E, F$. Kẻ đường kính $E J$ của đường tròn $(I)$. Gọi $d$ là đường thẳng qua $A$ song song với $B C$. Đường thẳng $J D$ cắt $d, B C$ lần lượt tại $L, H$.

(a) Chứng minh: $E, F, L$ thẳng hàng.

(b) $J A, J F$ cắt $B C$ lần lượt tại $M, K$. Chứng minh: $M H=M K$.

Bài 6. Tìm tất cả các số nguyên dương $x, y$ thỏa mãn phương trình: $3^{x}-y^{3}=1$

 

LỜI GIẢI

 

Bài 1. Cho ba số dương $a, b, c$ thỏa mãn điều kiện $\frac{a}{b+c}+\frac{b}{c+a}+\frac{c}{a+b}=2020$ Tính giá trị của biểu thức $P=\left(\frac{a^{2}}{b+c}+\frac{b^{2}}{c+a}+\frac{c^{2}}{a+b}\right):(a+b+c)$

Lời giải.

Ta có: $\left(\frac{a}{b+c}+\frac{b}{c+a}+\frac{c}{a+b}\right)(a+b+c)=2020(a+b+c)$

$\Leftrightarrow \frac{a^{2}}{b+c}+a+b+\frac{b^{2}}{c+a}+\frac{c^{2}}{a+b}+c=2020(a+b+c)$ $\Leftrightarrow \frac{a^{2}}{b+c}+\frac{b^{2}}{c+a}+\frac{c^{2}}{a+b}=2019(a+b+c)$ $\Leftrightarrow\left(\frac{a^{2}}{b+c}+\frac{b^{2}}{c+a}+\frac{c^{2}}{a+b}\right):(a+b+c)=2019$ Vạy $P=2019 .$

Bài 2.

a) Giải phương trình: $\sqrt{2 x^{2}+x+9}+\sqrt{2 x^{2}-x+1}=x+4$

b) Giải hệ phương trình: $\left\{\begin{array}{l}y^{2}-2 x y=8 x^{2}-6 x+1 \\ y^{2}=x^{3}+8 x^{2}-x+1\end{array}\right.$

Lời giải.

a) $\sqrt{2 x^{2}+x+9}+\sqrt{2 x^{2}-x+1}=x+4$ (1)

Đặt $\sqrt{2 x^{2}+x+9}=a(a>0)\left(\right.$ do $\left.2 x^{2}+x+9>0\right)$

và $\sqrt{2 x^{2}-x+1}=b(b>0)\left(\right.$ do $\left.2 x^{2}-x+1>0\right)$

Khi đó ta có: $a^{2}-b^{2}=2 x+8$

Thay vào phương trình ta có:

$a+b=\frac{a^{2}-b^{2}}{2} \Leftrightarrow 2(a+b)=(a-b)(a+b) \Leftrightarrow\left[\begin{array}{l}a+b=0 \\a-b=2\end{array}\right.$

  • TH1: $a+b=0 \Leftrightarrow\left\{\begin{array}{l}a=0 \\ b=0\end{array}\right.$ (Loại do $\left.a>0, b>0\right)$

  • TH2: $a-b=2$ khi đó ta có:

$\sqrt{2 x^{2}+x+9}-\sqrt{2 x^{2}-x+1}=2 $

$\Leftrightarrow \sqrt{2 x^{2}+x+9}=2+\sqrt{2 x^{2}-x+1} $

$\Leftrightarrow 2 x^{2}+x+9=4+2 x^{2}-x+1+4 \sqrt{2 x^{2}-x+1} $

$\Leftrightarrow x+2=2 \sqrt{2 x^{2}-x+1}$

$\Leftrightarrow\left\{\begin{array} { l }{ x \geq – 2 } \\{ x ^ { 2 } + 4 x + 4 = 8 x ^ { 2 } – 4 x + 4 }\end{array} \Leftrightarrow \left\{\begin{array}{l}x \geq-2 \\{\left[\begin{array}{l}x=0(n) \\x=\frac{8}{7}(n)\end{array}\right.}\end{array}\right.\right.$

Vậy $S=(0 ; \frac{8}{7})$

b) $\left\{\begin{array}{l}y^{2}-2 x y=8 x^{2}-6 x+1 \\ y^{2}=x^{3}+8 x^{2}-x+1(2)\end{array}\right.$

Từ $(2)$ ta có: $y^{2}=x^{3}+8 x^{2}-x+1$ thay vào $(1)$ ta có: $x^{3}+8 x^{2}-x+1-2 x y=8 x^{2}-6 x+1 \Leftrightarrow x^{3}-2 x y+5 x=0$ $\Leftrightarrow x\left(x^{2}-2 y+5\right)=0 \Leftrightarrow\left[\begin{array}{l}x=0 \\ x^{2}-2 y+5=0\end{array}\right.$

  • TH1: $x=0$ thay vào (2) ta có: $y^{2}=1 \Rightarrow y=\pm 1$

  • TH2: $x^{2}-2 y+5=0 \Leftrightarrow 2 y=x^{2}+5$ thay vào (2) ta có:

$4 y^{2}=4 x^{3}+32 x^{2}-4 x+4 $

$\Leftrightarrow\left(x^{2}+5\right)^{2}=4 x^{3}+32 x^{2}-4 x+4$

$\Leftrightarrow x^{4}-4 x^{3}-22 x^{2}+4 x+21=0 $

$\Leftrightarrow(x-7)(x+3)(x-1)(x+1)=0 $

$\Leftrightarrow\left[\begin{array}{l}x=7 \Rightarrow y=27 \\x=1 \Rightarrow y=3 \\x=-1 \Rightarrow y=3 \\x=-3 \Rightarrow y=7\end{array}\right.$

Vậy nghiệm của hệ phương trình là: $(-3 ; 7),(-1 ; 3),(0 ;-1),(0 ; 1),(1 ; 3)$, $(7 ; 27)$.

Bài 3. Cho tam giác nhọn $A B C(A B<B C<C A)$ nội tiếp đường tròn $(O)$. Từ $A$ kẻ đường thẳng song song với $B C$ cắt $(O)$ tại $A_{1}$. Từ $B$ kẻ đường thẳng song song với $A C$ cắt $(O)$ tại $B_{1}$. Từ $C$ kẻ đường thẳng song song với $A B$ cắt $(O)$ tại $C_{1}$. Chứng minh rằng các đường thẳng qua $A_{1}, B_{1}, C_{1}$ lần lượt vuông góc với $B C, C A, A B$ đồng quy.

Lời giải.

Gọi $H$ là trực tâm của tam giác $A B C$ và $O H$ cắt đường thẳng qua $A_{1}$, vuông góc với $B C$ ở điểm $K$. Gọi $M$ là trung điểm $A A_{1}$ thì $O M \perp A A_{1}$. Suy ra $O M \perp B C$.

Mặt khác, tứ giác $A H K A_{1}$ là hình thang vì $A H | A_{1} K$ nên ta có $O M$ là đường trung bình, kéo theo $O$ là trung điểm $H K$ hay nói cách khác, đường thẳng qua $A_{1}$, vuông góc với $B C$ sẽ đi qua điểm đối xứng với trực tâm $H$ của tam giác $A B C$ qua $O$.

Rõ ràng điểm này bình đẳng với $B, C$ nên hai đường qua $B_{1}, C_{1}$ lần lượt vuông góc với $C A, A B$ cũng đi qua $K$. Vì thế nên ta có các đường thẳng của đề bài đồng quy ở $K$.

Bài 4.

a) Cho 2 số thực $a, b$. Chứng minh rằng: $\frac{a^{2}+b^{2}}{2} \geq a b+\frac{(a-b)^{2}}{a^{2}+b^{2}+2}$

b) Cho hai số dương $a, b$ thỏa mãn điều kiện $a+b \leq 3$ Tìm giá trị nhỏ nhất của biểu thức: $Q=b-a+\frac{20}{a}+\frac{7}{b}$.

Lời giải.

a) Ta có: $\frac{a^{2}+b^{2}}{2} \geq a b+\frac{(a-b)^{2}}{a^{2}+b^{2}+2}$

$\Leftrightarrow \frac{(a-b)^{2}}{2} \geq \frac{(a-b)^{2}}{a^{2}+b^{2}+2} $

$\Leftrightarrow(a-b)^{2}\left(\frac{1}{2}-\frac{1}{a^{2}+b^{2}+2}\right) \geq 0 $

$\Leftrightarrow(a-b)^{2} \frac{a^{2}+b^{2}}{a^{2}+b^{2}+2} \geq 0 (đúng) $

b) Ta có: $a, b>0$ và $a \leq 3-b$

$Q=b-a+\frac{20}{a}+\frac{7}{b} \geq b-(3-b)+\frac{20}{3-b}+\frac{7}{b} $

$=(2 b-3)+\frac{20}{3-b}+\frac{7}{b} $

$=\left[5(3-b)+\frac{20}{3-b}\right]+\left(7 b+\frac{7}{b}\right)-18 $

$\geq 2 \sqrt{100}+2 \sqrt{49}-18=16$

Dấu “=” xảy ra khi và chỉ khi $a=2$ và $b=1$.

Bài 5. Đường tròn $(I)$ nội tiếp tam giác $A B C$ tiếp xúc với các cạnh $A B, B C, C A$ lần lượt tại $D, E, F$. Kẻ đường kính $E J$ của đường tròn $(I)$. Gọi $d$ là đường thẳng qua $A$ song song với $B C$. Đường thẳng $J D$ cắt $d, B C$ lần lượt tại $L, H$.

a) Chứng minh: $E, F, L$ thẳng hàng.

b) $J A, J F$ cắt $B C$ lần lượt tại $M, K$. Chứng minh: $M H=M K$.

Lời giải.

a) Ta có $J E$ là đường kính của $(I)$ nên $\angle J D E=90^{\circ}$ và $\triangle H D E$ vuông ở $D$. Chú ý rằng $B D=B E$, do cùng là tiếp tuyến kẻ từ $B$ đến $(I)$ nên $B D=B H$ (tính chất trung tuyến ứng với cạnh huyền). Do đó $\triangle B H D$ cân ở $B$.

Vị $A L / / B H$ nên $\triangle A D L$ và $\triangle B D H$ đồng dạng, kéo theo $\triangle A D L$ cân ở $A$ hay $A L=A D=A F$.

Vị $A L / / C E$ nên $\angle L A F=\angle F C E$, mà $\triangle A L F, \triangle C E F$ đều cân có các góc ở đỉnh bằng nhau nên chúng đồng dạng.

Suy ra $\angle A F L=\angle C F E$, kéo theo $L, F, E$ thẳng hàng.

b) Kéo dài $J F$ cắt $d$ ở $T$ thì tương tự câu $a$, ta có $T, D, E$ thẳng hàng và $A T=A D=$ $A F=A L$.

Theo định lý Thales với $d / / B C$ thì $\frac{A L}{M H}=\frac{A J}{J M}=\frac{A T}{M K}$, mà $A T=A L$ nên $M H=M K$.

Bài 6. Tìm tất cả các số nguyên dương $x, y$ thỏa mãn phương trình: $3^{x}-y^{3}=1$

Lời giải.

Ta có: $3^{x}-y^{3}=1 \Leftrightarrow y^{3}+1=3^{x}$

$\Leftrightarrow(y+1)\left(y^{2}-y+1\right)=3^{x} \Leftrightarrow\left\{\begin{array}{l}y+1=3^{m}(1) \\ y^{2}-y+1=3^{n}(2)\end{array}\right.$

  • TH1: $m=1 \Leftrightarrow y=2 \Rightarrow x=2$ (nhận).

  • TH2: $m \geq 2 \Leftrightarrow y=3^{m}-1$

Thế vào $(2):\left(3^{m}-1\right)^{2}-\left(3^{m}-1\right)+1=3^{n}$ $\Leftrightarrow 3^{n}=3^{2 m}-3^{m+1}+3=3^{m+1}\left(3^{m-1}-1\right)+3>3^{m+1} \Rightarrow n>m+1$

Ta lại có: $3=3^{n}-3^{2 m}+3^{m+1}=3^{m+1}\left(3^{n-m-1}-3^{m-1}+1\right) \vdots 3^{m+1} \Rightarrow m=0$ (vô lí).

Vậy phương trình có nghiệm $\left\{\begin{array}{l}x=2 \\ y=2\end{array}\right.$.

 

 

 

 

 

 

 

 

 

 

 

 

 

 

 

 

 

 

 

 

 

 

 

 

 

 

 

 

 

 

 

Đề thi Học kì 1 Toán 10 PTNK năm 2018 (CS2)

Bài 1. Giải các phương trình sau:
a) $\sqrt{7x+2} = 1 + \sqrt{4x+1}$
b) $\left| x^2-x-1 \right|+3= 2x$
Bài 2. Tìm $a$, $b$, $c$ biết parabol $(P):y= ax^2 + bx +c$ đi qua điểm $A(1;-1)$ và có đỉnh $I(-1; -5)$.
Bài 3. Tìm $m$ để phương trình $(x-1)\left( \sqrt{x+m}-1 \right) =0$ có hai nghiệm là độ dài các cạnh góc vuông của một tam giác vuông với cạnh huyền có độ dài bằng 3.
Bài 4. Cho hệ phương trình
$$\left\{ \begin{array}{l}
(1-2m)x +4y = 4m^2 + 4m +3 \
mx + 2(m-1)y=-m-2
\end{array} \right. \quad (I) $$
Chứng minh khi $m$ nhận giá trị bất kì trên $\mathbb{R}$, hệ $(I)$ luôn có nghiệm duy nhất. Giả sử $(x_0, 1)$ là nghiệm của hệ $(I)$. Tìm $x_0$.
Bài 5. Cho góc $\alpha$ thỏa $\tan \left( \alpha + \dfrac{\pi}{3} \right) = -\dfrac{ 3\sqrt{3}}{5}$. Tính giá trị của biểu thức:
$$ P=\dfrac{\cos ^3 \alpha + 2\sin \alpha \cdot \cos ^2 \alpha}{\sin ^2 \alpha \cdot \cos \alpha + \sqrt{3} \sin ^3 \alpha }$$
Bài 6. Cho tam giác $ABC$ có $AB=3a$, $AC=6a$, $BC=7a$.
a) Tính $\overrightarrow{AB} \cdot \overrightarrow{AC}$ và $\cos A$.
b) Gọi $M$, $N$ là hai điểm được xác định bởi $\overrightarrow{AM} = \dfrac{2}{3} \overrightarrow{AB}$, $\overrightarrow{AN} = -\dfrac{3}{2} \overrightarrow{AC}$, tính $MN$ theo $a$.
Bài 7. Trong mặt phẳng $Oxy$, cho tam giác $ABC$ có $A(1;3)$, $B(6;-2)$.
a) Tìm tọa độ điểm $C$ sao cho $G(1;1)$ là trọng tâm của tam giác $ABC$.
b) Tìm tọa độ chân đường vuông góc kẻ từ $M(4;3)$ đến đường thẳng $AB$.

Lời giải
Bài 1.
a) Nghiệm của phương trình: $x=2$.
b) $\left| x^2-x-1 \right| +3= 2x \\
\Leftrightarrow \left| x^2-x-1 \right| = 2x-3 \quad \left( x \ge \dfrac{3}{2} \right) \\
\Leftrightarrow \left[ \begin{array}{l}
x^2-x-1 = 2x-3 \\
x^2-x-1=3-2x
\end{array} \right. $

Từ đó suy ra nghiệm của phương trình: $x=2$ hoặc $x=\dfrac{\sqrt{17}-1}{2}$

Bài 2.
$P$ qua điểm $A(1;-1)$ nên $-1=a+b+c$.

$(P)$ có đỉnh $I(-1;-5)$ nên $-5=a-b+c$ và $-\dfrac{b}{2a}= -1$.

Từ đó suy ra $P: y= x^2 + 2x-4$.

Bài 3. Điều kiện: $x\ge -m$

Từ phương trình suy ra: $\left[ \begin{array}{l}
x= 1 \\
x= 1-m
\end{array} \right. $

Để hai nghiệm là độ dài các cạnh góc vuông của tam giác vuông có cạnh huyền bằng 3 thì: $1^2 + (1-m)^2 =3^2 \Rightarrow \left[ \begin{array}{l}
m= 1+2\sqrt{2} \\
m= 1-2\sqrt{2} \quad \text{(loại vì } x\ge -m)
\end{array} \right. $

Vậy $m=1+2\sqrt{2}$.

Bài 4. $D= \left| \begin{array}{*{20}{c}}
{1-2m}&{4}\\
{m}&{2(m-1)}
\end{array} \right| = (1-2m)(2m-2)-4m = -4m^2+2m-2 $

$D_x = \left| \begin{array}{*{20}{c}}
{4m^2+4m+3}&{4}\\
{-m-2}&{2(m-1)}
\end{array} \right| = 8m^3+2m+2 $

$D_y= \left| \begin{array}{*{20}{c}}
{1-2m}&{4m^2+4m+3}\\
{m}&{-m-2}
\end{array} \right| = -4m^3-2m^2-2 $

$D=0 \Leftrightarrow -4m^2+2m-2 =0$ (vô nghiệm).

Suy ra $D \ne 0$ với mọi $m \in \mathbb{R}$

Vậy với mọi $m \in \mathbb{R}$ thì hệ $(I)$ luôn có nghiệm duy nhất.

Khi $(x_0;1)$ là nghiệm của hệ $(I)$ thì $y=\dfrac{D_y}{D}=\dfrac{-4m^3-2m^2-2}{-4m^2+2m-2}=1 \Leftrightarrow m=0$

Khi đó $x=\dfrac{D_x}{D}= \dfrac{2}{-2}=-1$

Bài 5. $\tan\left( \alpha + \dfrac{\pi}{3} \right) =\dfrac{\tan \alpha + \tan \dfrac{\pi}{3}}{1-\tan \alpha \cdot \tan \dfrac{\pi}{3}} = -\dfrac{3\sqrt{3}}{5} \Leftrightarrow \tan \alpha = 2\sqrt{3}$

$P=\dfrac{\cos ^3 \alpha + 2\sin \alpha \cdot \cos ^2 \alpha}{\sin ^2 \alpha \cdot \cos \alpha + \sqrt{3} \sin ^3 \alpha } = \dfrac{1+2\tan \alpha}{\tan ^2 \alpha + \sqrt{3} \tan ^3 \alpha} = \dfrac{1+4\sqrt{3}}{84}$
Bài 6.
a) $\overrightarrow{AB} \cdot \overrightarrow{AC} = -2a^2$; $\cos A = \dfrac{-2a^2}{3a\cdot 6a}= -\dfrac{1}{9}$
b) $\overrightarrow{MN} = \overrightarrow{AN} – \overrightarrow{AM} = -\dfrac{3}{2} \overrightarrow{AC} – \dfrac{2}{3} \overrightarrow{AB} \Rightarrow MN^2=\overrightarrow{MN}^2 = \ldots = 81a^2 \Rightarrow MN = 9a$

Bài 7.
a) $C(-4;2)$
b) Gọi chân đường vuông góc hạ từ $M$ đến $AB$ là $H(x,y)$

Ta có: $\left\{ \begin{array}{l}
\overrightarrow{MH} \bot \overrightarrow{AB} \\
\overrightarrow{AH} \parallel \overrightarrow{AB}
\end{array} \right. $

Từ đó suy ra: $H\left( \dfrac{5}{2}; \dfrac{3}{2} \right) $

Đề thi Học kì 1 Toán 10 PTNK năm 2017 (CS2)

Đề và lời giải: Thầy Nguyễn Tấn Phát

Bài 1. Giải các phương trình sau:
a) $(x+2)\sqrt{x^2-5}=x^2-4$
b) $x^2+8x+|x+4|+14=0$
Bài 2. Tìm $a$, $b$, $c$ biết hàm số $y=ax^2+bx+c$ có đồ thị được cho như hình sau.

Bài 3. Tìm $m$ để phương trình $(m-1)^2x^2 – 4(m+1)x+3=0$ có hai nghiệm, trong đó có một nghiệm gấp 3 lần nghiệm còn lại.
Bài 4. Tìm số nguyên $m$ sao cho hệ $\left{ \begin{array}{l}
mx-y=1 \
x+4(m+1)y=4m
\end{array} \right. $ có nghiệm duy nhất và là nghiệm nguyên.
Bài 5. Tính giá trị của biểu thức $P=\dfrac{16\cos ^3 a – \sin ^3 a + 5\cos a}{9\cos a + \sin ^3 a}$ khi $\tan a =3$.

Bài 6. Cho ba vectơ $\overrightarrow{a}$, $\overrightarrow{b}$, $\overrightarrow{c}$ bất kì. Xét tính đúng, sai của các mệnh đề sau:
a) $\left[ \left( \overrightarrow{a} \cdot \overrightarrow{b} \right) \overrightarrow{c} – \left( \overrightarrow{a} \cdot \overrightarrow{c} \right) \overrightarrow{b} \right] $ vuông góc với $\overrightarrow{a}$
b) $\left( \overrightarrow{a}\cdot \overrightarrow{b} \right) \overrightarrow{c} = \left( \overrightarrow{b} \cdot \overrightarrow{c} \right) \overrightarrow{a}$
Bài 7. Cho $\overrightarrow{u}= (1;-2)$, $\overrightarrow{v} = (x;y)$. Tìm $x$, $y$ sao cho $\overrightarrow{u}$, $\overrightarrow{v}$ cùng phương và $\overrightarrow{u} \cdot \overrightarrow{v}=-\dfrac{13}{2}$. Tính $|\overrightarrow{v}|$.
Bài 8. Cho tam giác $ABC$ với $A(-3;6)$, $B(1;-2)$, $C(6;3)$. Tìm tọa độ tâm $I$ và bán kính đường tròn ngoại tiếp của tam giác $ABC$.
Bài 9. Cho các điểm $M(-1;2m+3)$, $N(-4; 5m)$ và $P(-3; 3m+2)$. Tìm điều kiện cần và đủ của $m$ để $M$, $N$, $P$ là ba đỉnh của một tam giác. Khi đó chứng minh $\angle NMP$ là góc nhọn.

Lời giải
Bài 1.
a) Nghiệm của phương trình $x=\dfrac{9}{4}$
b) Phương trình tương đương: $(x+4)^2 + |x+4| -2=0$. Đặt $t= |x+4|$, phương trình trở thành $t^2 +t-2=0$.

Từ đó giải được tập nghiệm của phương trình là $S=\left\{ -3;-5 \right\} $
Bài 2. $(P):y=x^2 -4x+2$

Bài 3. Để phương trình có hai nghiệm phân biệt $x_1$, $x_2$ thì $\left\{ \begin{array}{l}
m \ne 1 \\
\Delta = m^2 +14m+1 >0
\end{array} \right. $

Theo Viete, ta có: $\left\{ \begin{array}{l}
x_1+x_2 = \dfrac{4(m+1)}{(m-1)^2} \quad (2) \\
x_1x_2= \dfrac{3}{(m-1)^2} \quad (3)
\end{array} \right. $

Vì nghiệm này gấp ba nghiệm kia nên từ $(2)$, ta tìm được $x_1$, $x_2$ theo $m$, sau đó thay vào $(3)$ giải được $m=0$ (nhận)

Vậy $m=0$

Bài 4. Để hệ có nghiệm duy nhất thì $D \ne 0 \Leftrightarrow m \ne -\dfrac{1}{2}$.

Tính $D_x$, $D_y$, suy ra $x=\dfrac{4}{2m+1}$, $y=\dfrac{2m-1}{2m+1}$.

Để nghiệm nguyên thì $2m+1 \in U(4)$ và $2m-1 \, \vdots \, 2m+1$. Từ đó suy ra $m \in \left\{ -1; 3 \right\} $
Bài 5. Vì $\cos a \ne 0$ nên chia cả tử và mẫu của $P$ cho $\cos ^3 a$, ta được:
$$P= \dfrac{16-\tan ^3 a + 5 \left( \tan ^2 a +1 \right) }{9 \left( \tan ^2 a +1 \right) + \tan ^3 a} = \dfrac{1}{3}$$
Bài 6.
a) Xét tích vô hướng:
$\left[ \left( \overrightarrow{a} \cdot \overrightarrow{b} \right) \overrightarrow{c} – \left( \overrightarrow{a} \cdot \overrightarrow{c} \right) \overrightarrow{b} \right] \overrightarrow{a} \\= |\overrightarrow{a}|\cdot |\overrightarrow{b} | \cdot \cos (\overrightarrow{a} , \overrightarrow{b} ) \cdot |\overrightarrow{c}|\cdot |\overrightarrow{a} | \cdot \cos (\overrightarrow{c} , \overrightarrow{a} ) – |\overrightarrow{c}|\cdot |\overrightarrow{a} | \cdot \cos (\overrightarrow{c} , \overrightarrow{a} ) \cdot |\overrightarrow{a}|\cdot |\overrightarrow{b} | \cdot \cos (\overrightarrow{a} , \overrightarrow{b} ) = 0$
Suy ra $\left[ \left( \overrightarrow{a} \cdot \overrightarrow{b} \right) \overrightarrow{c} – \left( \overrightarrow{a} \cdot \overrightarrow{c} \right) \overrightarrow{b} \right] $ vuông góc với $\overrightarrow{a}$
b) $\left( \overrightarrow{a}\cdot \overrightarrow{b} \right) \overrightarrow{c}$ cùng phương với $\overrightarrow{c}$; $\left( \overrightarrow{b} \cdot \overrightarrow{c} \right) \overrightarrow{a}$ cùng phương với $\overrightarrow{a}$

Xét trường hợp $\overrightarrow{a}$ không cùng phương với $\overrightarrow{c}$ thì mệnh đề trên sai. Vậy mệnh đề trên sai.

Bài 7. $\overrightarrow{v}=\left( -\dfrac{13}{10} ; \dfrac{13}{5} \right) \Rightarrow |\overrightarrow{v} | = \dfrac{13\sqrt{5}}{10}$
Bài 8. $I(1;3)$, $R=5$
Bài 9. Để $MNP$ là tam giác thì $m \ne 1$

$\cos NMP = \dfrac{2+(m-1)^2}{\sqrt{1+(m-1)^2} \sqrt{4+(m-1)^2}} >0, \; \forall m$ nên $\angle NMP$ là góc nhọn.

Đề thi Học kì 1 Toán 10 PTNK năm 2019 (CS1)

Đề và lời giải: Thầy Nguyễn Tấn Phát

Bài 1. (1 điểm) Tìm $m$ để phương trình $\dfrac{m^2x+m}{x-1}=1$ có đúng một nghiệm.
Bài 2. (2 điểm) Giải các phương trình sau:
a) $4x-\left| 3x-2 \right| =x^2$
b) $\left( x^2 +x-2 \right) \left( \sqrt{5x-1}-7+2x \right) =0$
Bài 3. (1 điểm) Cho parabol $(P): y=ax^2+bx+c$. Tìm $a$, $b$, $c$ biết điểm $B(-1;4)$ thuộc $(P)$ và $S(0;3)$ là đỉnh của parabol.
Bài 4. (1,5 điểm) Cho hệ phương trình $\left\{ \begin{array}{l}
2mx-(m+1)y=m+1 \
(m-2)x-\dfrac{m}{2}y=-\dfrac{m}{2}-2
\end{array} \right. $
a) Tìm $m$ để hệ phương trình có nghiệm.
b) Tìm nghiệm $\left( x_0; y_0 \right) $ của hệ thỏa $x_0-y_0=-2$
Bài 5. (0,5 điểm) Rút gọn: $P=\dfrac{\sin \left( x+ \dfrac{\pi}{2} \right) + 2\cos (x+ \pi)}{\cos (\pi -x )}$.
Bài 6. (2 điểm) Hình bình hành $ABCD$ có $AB=a$, $AD=a\sqrt{3}$ và $\angle BAD = 30^\circ $
a) Tính $\overrightarrow{AB} \cdot \overrightarrow{AD}$ và độ dài đoạn $AC$.
b) Gọi $DE$ là đường cao của tam giác $ABD$ ($E$ thuộc đường thẳng $AB$). Tính $\overrightarrow{AB}\cdot \overrightarrow{AE}$ và độ dài đoạn $DE$.
Bài 7. (2 điểm) Trong mặt phẳng $Oxy$, cho $A(6;-2)$, $B(3;-1)$, $C(9;7)$.
a) Chứng minh $ABC$ là tam giác vuông và tìm $I$ thuộc trục tung sao cho $\overrightarrow{IB} \cdot \overrightarrow{AB} =10$.
b) Tính độ dài đoạn $AG$ với $G$ là trọng tâm tam giác $ABC$. Tìm điểm $K$ thuộc đường thẳng $d: y=x$ sao cho $\left| \overrightarrow{KB} + \overrightarrow{KC} \right| = 2\sqrt{5}$

Lời giải
Bài 1. (1 điểm)
a) Điều kiện: $x\ne 1$

Phương trình $(1) \Rightarrow \left( m^2-1 \right) x=-m-1 \quad (2)$

Phương trình $(1)$ có đúng một nghiệm khi và chỉ khi phương trình $(2)$ có đúng một nghiệm khác 1

$\Leftrightarrow \left\{ \begin{array}{l}
m^2-1 \ne 0 \\
x= \dfrac{-m-1}{m^2-1} \ne 1
\end{array} \right. \Leftrightarrow \left\{ \begin{array}{l}
m\ne 1 \\
m\ne -1 \\
m \ne 0
\end{array} \right. $

Bài 2. (2 điểm)
a)

  • Nếu $x\ge \dfrac{2}{3}$ ta có phương trình:
    $$4x-3x+2=x^2 \Leftrightarrow x^2-x-2=0 \Leftrightarrow \left[ \begin{array}{l}
    x=-1 \quad \text{(loại)} \\
    x=2 \quad \text{(nhận)}
    \end{array} \right. $$
  • Nếu $x < \dfrac{2}{3}$ ta có phương trình:
    $$4x+3x-2=x^2 \Leftrightarrow x^2-7x+2=0 \Leftrightarrow \left[ \begin{array}{l}
    x=\dfrac{7+\sqrt{41}}{2} \quad \text{(loại)} \\
    x=\dfrac{7-\sqrt{41}}{2} \quad \text{(nhận)}
    \end{array} \right. $$

Vậy phương trình có hai nghiệm $x=2$, $x=\dfrac{7-\sqrt{41}}{2}$
b) Điều kiện: $x\ge \dfrac{1}{5}$

$\left( x^2 +x-2 \right) \left( \sqrt{5x-1}-7+2x \right) =0 \Leftrightarrow \left[ \begin{array}{l}
x^2+x-2=0 \quad (2) \\
\sqrt{5x-1}=7-2x \quad (3)
\end{array} \right. $

$(1)\Leftrightarrow \left[ \begin{array}{l}
x=1 \quad \text{(nhận)} \\
x=-2 \quad \text{(loại)}
\end{array} \right. $

$(2)\Leftrightarrow \left\{ \begin{array}{l}
7-2x \ge 0 \\
5x-1 = 49 -28x+4x^2
\end{array} \right. \Leftrightarrow \left\{ \begin{array}{l}
x \le \dfrac{7}{2} \\
4x^2 – 33x + 50=0
\end{array} \right. \\
\Leftrightarrow \left\{ \begin{array}{l}
x\le \dfrac{7}{2} \\
\left[ \begin{array}{l}
x=2 \\
x=\dfrac{25}{4}
\end{array} \right.
\end{array} \right. \Leftrightarrow x=2 \quad \text{(nhận)}$
Vậy phương trình có 2 nghiệm $x=1$, $x=2$.

Bài 3. (1 điểm) $B(-1;4) \in (P) \Rightarrow 4=a-b+c$

$S(0;3)$ là đỉnh nên $3=c$ và $-\dfrac{b}{2a}=0 \Rightarrow b=0 \Rightarrow a=1$

Vậy $a=1$, $b=0$, $c=3$.

Bài 4. (1,5 điểm)
a) $D=\left| \begin{array}{*{20}{c}}
{2m}&{-(m+1)}\\
{m-2}&{-\dfrac{m}{2}}
\end{array} \right| = -m-2$

$D_x=\left| \begin{array}{*{20}{c}}
{m+1}&{-(m+1)}\\
{-\dfrac{m}{2}-2}&{-\dfrac{m}{2}}
\end{array} \right| = (m+1)(-m-2)$

$D_y=\left| \begin{array}{*{20}{c}}
{2m}&{m+1}\\
{m-2}&{-\dfrac{m}{2}-2}
\end{array} \right| = (-m-2)(2m-1)$
Hệ có nghiệm duy nhất khi và chỉ khi $D \ne 0 \Leftrightarrow -m-2 \ne 0 \Leftrightarrow m \ne -2$

Hệ có vô số nghiệm khi và chỉ khi $D=D_x=D_y=0\Leftrightarrow m=-2$

Khi đó hệ có vô số nghiệm là $(x,y)$ thỏa $-4x+y=-1$

Vậy hệ có nghiệm với mọi giá trị của $m$.
b)

  • TH1. Hệ có một nghiệm duy nhất khi và chỉ khi $D \ne 0\ne m \ne -2$Khi đó $x_0=m+1$, $y_0=2m-1$

    $x_0-y_0=-2 \Leftrightarrow m+1-2m+1=-2 \Leftrightarrow m=4$ (nhận)

    Khi đó nghiệm của hệ là $(5;7)$.

  • TH2. Hệ có vô số nghiệm khi và chỉ khi $D=D_x=D_y=0 \Leftrightarrow m=-2$Khi đó hệ có vô số nghiệm $\left( x_0;y_0 \right) $ thỏa $-4x_0+y_0=-1$

    Khi đó ta có hệ phương trình $\left\{ \begin{array}{l}
    x_0-y_0=-2 \\
    -4x_0+y_0=-1
    \end{array} \right. \Leftrightarrow \left\{ \begin{array}{l}
    x_0=1 \\
    y_0=3
    \end{array} \right. $

    Hệ có nghiệm là $(1;3)$.

Bài 5. (0,5 điểm) $\sin \left( x+ \dfrac{\pi}{2} \right) =\cos x$, $\cos (x+ \pi) = -\cos x$, $\cos (\pi -x) = – \cos x$

$P=\dfrac{\cos x – 2\cos x}{-\cos x} = 1$
Bài 6. (2 điểm)
a) $\overrightarrow{AB} \cdot \overrightarrow{AD} = AB\cdot AD \cdot \cos BAD = \dfrac{3}{2} a^2$

$\overrightarrow{AC} = \overrightarrow{AB} + \overrightarrow{AD}$

$AC^2 = \left( \overrightarrow{AB} + \overrightarrow{AD} \right) ^2 = AB^2 + AD^2 + 2\overrightarrow{AB} \cdot \overrightarrow{AD} = a^2 + 3a^2 + 3a^2 = 7a^2$

$\Rightarrow AC=\sqrt{7}a$
b) Đặt $\overrightarrow{AE} = x\cdot \overrightarrow{AB}$

$\overrightarrow{DE} = \overrightarrow{AE} – \overrightarrow{AD} = x\overrightarrow{AB}- \overrightarrow{AD}$

$DE \bot AB \Leftrightarrow \overrightarrow{DE} \bot \overrightarrow{AB} \Leftrightarrow \overrightarrow{DE} \cdot \overrightarrow{AB} =0 \\
\Leftrightarrow \left( x\overrightarrow{AB} – \overrightarrow{AD} \right) \cdot \overrightarrow{AB} = 0 \\
\Leftrightarrow xAB^2 – \overrightarrow{AB} \cdot \overrightarrow{AD} =0 \Leftrightarrow xa^2 – \dfrac{3}{2}a^2 =0 \Leftrightarrow x=\dfrac{3}{2}$

Vậy $\overrightarrow{AE} = \dfrac{3}{2} \overrightarrow{AB}$

$\overrightarrow{AB} \cdot \overrightarrow{AE} = \overrightarrow{AB} \cdot \left( \dfrac{3}{2} \overrightarrow{AB} \right) = \dfrac{3}{2} AB^2 = \dfrac{3}{2} a^2$

$DE^2 = \left( \dfrac{3}{2} \overrightarrow{AB} – \overrightarrow{AD} \right) ^2 = \dfrac{9}{4}AB^2 + AD^2 – 3\overrightarrow{AB} \cdot \overrightarrow{AD} = \dfrac{9}{4}a^2 + 3a^2 – \dfrac{9}{2}a^2 = \dfrac{3}{4}a^2 \\
\Rightarrow DE = \dfrac{\sqrt{3}}{2}a$

Hoặc cách khác.

Tam giác $ABE$ vuông tại $E$ có $\angle DAE = 30^\circ $ nên $\sin 30^\circ = \dfrac{DE}{AD} \Rightarrow DE=AD \cdot \sin 30^\circ = a\sqrt{3}\cdot \dfrac{1}{2} = \dfrac{a\sqrt{3}}{2}$

Bài 7. (2 điểm)
a) $\overrightarrow{AB}(-3;1)$, $\overrightarrow{AC}(3;9)$

$\overrightarrow{AB} \cdot \overrightarrow{AC} = -3\cdot 3 + 1\cdot 9 =0 \Rightarrow \overrightarrow{AB} \bot \overrightarrow{AC} \Rightarrow \triangle ABC$ vuông tại $A$.

Đặt $I(0;i)$

$\overrightarrow{IB}(3;-1-i)$, $\overrightarrow{AB}(-3;1)$

$\overrightarrow{IB} \cdot \overrightarrow{AB} =10 \Leftrightarrow 3\cdot (-3) + (-1-i)\cdot 1= 10 \Leftrightarrow i=-20$

Vậy $I(0;-20)$.
b) $G\left( 6; \dfrac{4}{3} \right) $

$\overrightarrow{AG}\left( 0; \dfrac{10}{3} \right) \Rightarrow AG= \dfrac{10}{3}$

Đặt $K(k;k)$

$\overrightarrow{KB}(3-k;-1-k)$, $\overrightarrow{KC}(9-k;7-k)$

$\overrightarrow{KB}+ \overrightarrow{KC} = (12-2k;6-2k)$

$\left| \overrightarrow{KB} + \overrightarrow{KC} \right| = 2\sqrt{5} \Leftrightarrow (12-2k)^2 + (6-2k)^2 =20 \\
\Leftrightarrow k^2-9k+20=0 \Leftrightarrow \left[ \begin{array}{l}
k=4 \\
k=5
\end{array} \right. $

Vậy $K(4;4)$ hoặc $K(5;5)$.

Đề thi Học kì 1 Toán 10 PTNK năm 2020 (CS2)

Bài 1. (2 điểm) Giải các phương trình:
a) $\dfrac{x^2 – 3x -4}{\sqrt{3-x}}=0$
b) $\sqrt{x+2}= \sqrt{2x+5} – \sqrt{3-x}$
Bài 2. (1 điểm) Tìm tọa độ đỉnh $I$ của parabol $(P): y= ax^2 + bx+ c \ (a \ne 0)$, biết parabol $(P)$ cắt trục hoành tại hai điểm có hoành độ lần lượt bằng 2 và 8, cắt trục tung tại điểm có tung độ bằng 8.
Bài 3. (1 điểm) Tìm $m$ để phương trình $\dfrac{x(2-x)}{\sqrt{2-x}} = \left( m^2 +1 \right) \sqrt{2-x}$ có nghiệm.
Bài 4. (1 điểm) Tìm $m$ để hệ phương trình $\left\{ \begin{array}{l}
(m+1)x-2y =m-1 \
m^2x-y = m^2 + 2m
\end{array} \right. $ có nghiệm duy nhất $\left( x_0; y_0 \right) $. Xác định một hệ thức liên hệ giữa $x_0$ và $y_0$ mà không phụ thuộc vào $m$.
Bài 5. (1 điểm) Cho góc $a$ thỏa $\tan \left( a + \dfrac{\pi}{2} \right) = -\sqrt{3}$. Tính giá trị của biểu thức:
$$P=\dfrac{\sin ^6 a + \cos ^6 a + 2\sin ^3 a \cdot \cos ^3 a}{\sin ^5 a \cdot \cos ^3 a + \sin ^3 a \cdot \cos ^5 a}$$
Bài 6. (2 điểm) Cho tam giác $ABC$ nhọn có độ dài cạnh $AB=5$. Gọi $H$ là chân đường cao hạ từ $A$ và $BH=3$, $CH=6$.
a) Tính $\overrightarrow{BA} \cdot \overrightarrow{BC}$ và độ dài $AC$.
b) Gọi $M$ là trung điểm của $AH$. Tính $\overrightarrow{MB} \cdot \overrightarrow{MC}$.
Bài 7. (2 điểm) Trong mặt phẳng $Oxy$, cho tam giác $ABC$ có $A(1;2)$, $B(-1;5)$, $C(3;2)$.
a) Tìm tọa độ trọng tâm $G$ và điểm $I$ thỏa $2\overrightarrow{IA} + 3\overrightarrow{IB} = 4\overrightarrow{IC}$.
b) Tìm tọa độ điểm $D$ biết $ABCD$ là hình thang có đáy $AB = \dfrac{3}{8}CD$.

Lời giải
Bài 1.
a) Điều kiện: $x<3$
$\dfrac{x^2 – 3x -4}{\sqrt{3-x}}=0 \Leftrightarrow (x+1)(x-4)=0 \Leftrightarrow \left[ \begin{array}{l}
x=-1 \quad (n) \\
x=4 \quad \ (l)
\end{array} \right. $
Vậy $S=\left\{ -1 \right\} $

b) $\sqrt{x+2}= \sqrt{2x+5} – \sqrt{3-x} \quad (1)$
Điều kiện: $-2 \le x \le 3$
$(1) \Leftrightarrow \sqrt{x+2} + \sqrt{3-x} = \sqrt{2x+5} \\
\Leftrightarrow 5 + 2\sqrt{-x^2+x+6} = 2x+5 \\
\Leftrightarrow \sqrt{-x^2+x+6} = x \quad (x\ge 0) \\
\Leftrightarrow -x^2 + x+6 = x^2 \\
\Leftrightarrow \left[ \begin{array}{l}
x=-\dfrac{3}{2} \quad (l) \\
x=2 \hspace{0.8cm} (n)
\end{array}\right.$

Vậy $S=\left\{ 2 \right\} $

Bài 2. (1 điểm) Theo đề ta có $(2;0)$, $(8;0)$, $(0;8)$ thuộc $(P)$ nên ta có hệ:

$\left\{ \begin{array}{l}
4a + 2b + c=0 \\
64a + 8b + c =0 \\
c=8
\end{array} \right. \Leftrightarrow \left\{ \begin{array}{l}
a=\dfrac{1}{2} \\
b=-5 \\
c=8
\end{array} \right. $

Suy ra $(P): y=\dfrac{1}{2}x^2 -5x+8$

Vậy tọa độ đỉnh $I$ của $(P)$ là $I\left( 5 ; -\dfrac{9}{2} \right) $
Bài 3. (1 điểm) ĐKXĐ: $x<2$
$\dfrac{x(2-x)}{\sqrt{2-x}} = \left( m^2 +1 \right) \sqrt{2-x} \Leftrightarrow x(2-x) = (m^2+1) (2-x)$
$\Leftrightarrow \left[ \begin{array}{l}
x=2 \ (l)\\
x=m^2 +1
\end{array}\right.$
Phương trình có nghiệm khi và chỉ khi $m^2+1 <2 \Leftrightarrow m^2 <1 \Leftrightarrow -1 < m <1$
Vậy khi $-1<m<1$ thì phương trình có nghiệm $x=m^2+1$.
Bài 4. (1 điểm)
Ta có:
$D=\left| \begin{array}{*{20}{c}}
{m+1}&{-2}\\
{m^2}&{-1}
\end{array}\right| = 2m^2 -m-1=(m-1)(2m+1)$

$D_x = \left| \begin{array}{*{20}{c}}
{m-1} & {-2}\\
{m^2 +2m} & {-1}
\end{array}\right| = 2m^2 +3m+1 = (2m+1)(m+1)$

$D_y=\left| \begin{array}{*{20}{c}}
{m+1} & {m-1}\\
{m^2} & {m^2 +2m}
\end{array}\right| = 4m^2 +2m = 2m(2m+1)$

Hệ phương trình có nghiệm duy nhất khi và chỉ khi $D\ne 0 \Leftrightarrow \left\{ \begin{array}{l}
m\ne 1\\
m\ne -\dfrac{1}{2}
\end{array}\right.$

Ta có: $\left\{ \begin{array}{l}
x_0 = \dfrac{D_x}{D} = \dfrac{m+1}{m-1}\\\\
y_0= \dfrac{D_y}{D} = \dfrac{2m}{m-1}
\end{array}\right. $

Ta có: $y_0 -x_0 = \dfrac{2m-m-1}{m-1} = 1$
Bài 5. (1 điểm) Ta có: $\tan \left(\alpha + \dfrac{\pi}{2}\right) = -\sqrt{3} \Leftrightarrow \tan \alpha = \dfrac{1}{\sqrt{3}}$

ĐKXĐ: $\cos \alpha \ne 0$

Chia cả tử và mẫu của $P$ cho $\cos \alpha ^8$ ta được:

$P=\dfrac{\sin ^6 a + \cos ^6 a + 2\sin ^3 a \cdot \cos ^3 a}{\sin ^5 a \cdot \cos ^3 a + \sin ^3 a \cdot \cos ^5 a}$

$=\dfrac{ \tan \alpha ^6 \cdot (\tan \alpha ^2 +1) + \tan \alpha ^2 +1 + 2\tan \alpha ^3 \cdot (\tan \alpha ^2 +1)}{\tan \alpha ^5 + \tan \alpha ^3}$

$=\dfrac{28+8\sqrt{3}}{3\sqrt{3}}$
Bài 6. (2 điểm)
a) Tam giác $ABH$ vuông tại $H$ nên $AH^2 = AB ^2 – BH^2 =16 \Rightarrow AH=4$

Tam giác $ACH$ vuông tại $H$ nên $AC^2 = AH^2 + CH^2 = 52 \Rightarrow AC=2\sqrt{13}$

Ta có: $CA^2 = \overrightarrow{CA}^2 = (\overrightarrow{BA} – \overrightarrow{BC} )^2 = BA^2 + BC^2 -2\overrightarrow{BA}\cdot \overrightarrow{BC}$

$\Rightarrow 52 = 25+ 81 – 2 \overrightarrow{BA} \cdot \overrightarrow{BC}$
$\Rightarrow \overrightarrow{BA} \cdot \overrightarrow{BC} = 27$
b) Tam giác $MBH$ vuông tại $H$ có: $MB^2 = MH^2 + BH^2 = 13 \Rightarrow MB =\sqrt{13}$

Tam giác $MCH$ vuông tại $H$ có: $MC^2 = MH^2 + CH^2 = 40 \Rightarrow MC = 2\sqrt{10}$

Ta có: $ CB^2 = (\overrightarrow{MB} – \overrightarrow{MC})^2 = MB^2 + MC^2 – 2\overrightarrow{MB} \cdot \overrightarrow{MC}$

$\Rightarrow 81 = 13 + 40 – 2\overrightarrow{MB} \cdot \overrightarrow{MC} \Rightarrow \overrightarrow{MB} \cdot \overrightarrow{MC} = -14$
Bài 7. (2 điểm)
a) Gọi $G(x_G; y_G)$

Ta có: $\left\{ \begin{array}{l}
x_G = \dfrac{x_A+x_B+x_C}{3} = 1\\
y_G = \dfrac{y_A + y_B + y_C}{3} =3
\end{array}\right. \Rightarrow G(1;3)$

Gọi $I(x_I; y_I)$

Khi đó $\overrightarrow{IA} = (1-x_I; 2-y_I)$, $\overrightarrow{IB} = (-1-x_I; 5-y_I)$, $\overrightarrow{IC} = (3-x_I; 2-y_I)$

Ta có: $2\overrightarrow{IA} + 3 \overrightarrow{IB} = 4\overrightarrow{IC} \Rightarrow \left\{ \begin{array}{l}
2(1-x_I) + 3(-1-x_I) = 4(3-x_I)\\
2(2-y_I) + 3(5-y_I) = 4(2-y_I)
\end{array}\right.$
$\Rightarrow \left\{ \begin{array}{l}
x_I = -13\\
y_I = 11
\end{array}\right.$

Vậy $I(-13; 11)$.
b) Gọi $D(x_D; y_D)$ khi đó $\overrightarrow{DC} = (3-x_D ; 2-y_D)$, $\overrightarrow{AB} = (-2; 3)$

Ta có: $AB // CD$ và $AB = \dfrac{3}{8}CD$

$ \Rightarrow \overrightarrow{AB} = \dfrac{3}{8} \overrightarrow{DC} \Rightarrow \left\{ \begin{array}{l}
-2 = \dfrac{3}{8} (3-x_D)\\
3 = \dfrac{3}{8} (2-y_D)
\end{array}\right. \Rightarrow \left\{ \begin{array}{l}
x_D = \dfrac{25}{3}\\
y_D = -6
\end{array}\right.$

Vậy $D\left(\dfrac{25}{3}; -6\right)$

Đề thi Học kì 1 Toán 10 PTNK năm 2020 (CS1)

Đề thi và đáp án HK1 môn toán 10 trường PTNK (CS1)

Năm học 2020 – 2021

Thực hiện: Thầy Nguyễn Tấn Phát – GV PTNK

Bài 1. (2 điểm) Giải các phương trình:
a) $\dfrac{{{x^4} – 10{x^2} + 9}}{{\sqrt {x – 2} }} = 0$
b) $x\sqrt {{x^2} – x + 3} = x\left( {x – 6} \right)$
Bài 2. (1 điểm) Tìm $m$ để phương trình $\dfrac{1}{x} + \dfrac{{m + x}}{{x – 1}} = 1$ có nghiệm duy nhất.
Bài 3. (1 điểm) Chứng minh
$$\left[ {\cos 2\pi – \cos \left( {2\pi + x} \right)} \right]\left[ {1 + {{\tan }^2}\left( {\frac{\pi }{2} – x} \right)} \right] = \frac{1}{{1 + \cos x}}$$
Bài 4. (1 điểm) Cho hệ phương trình $\left\{ \begin{array}{l}
mx – \left( {m + 1} \right)y = 1\
\left( {2 – m} \right)x + \left( {m – 3} \right)y = 3 – 2m
\end{array} \right.$ ($m$ là tham số).
a) Tìm $m$ để hệ có nghiệm duy nhất $\left( x_0; y_0 \right) $.
b) Chứng minh $x_0^2 – y_0^2 – 2{x_0} = – 1$
Bài 5. (1 điểm) Gọi $(P)$ là đồ thị của hàm số $y = {x^2} + 2x – m$. Biết $(P)$ cắt trục tung tại điểm có tung độ là 4. Tìm m và tọa độ đỉnh của $(P)$.
Bài 6. (2 điểm) Cho hình bình hành ABCD có $AD = a$, $AB = 2a$ và $\widehat {DAB} = 120^\circ $.
a) Tính $\overrightarrow{DA} \cdot \overrightarrow{AB}$. Chứng minh $AB^2 – AD^2 = \overrightarrow {AC} \cdot \overrightarrow {DB} $
b) Gọi $H$ là hình chiếu vuông góc của $A$ trên $DB$. Tính $\overrightarrow{DH} \cdot \overrightarrow{DA}$.
Bài 7. (2 điểm) Trong mặt phẳng $Oxy$, cho tam giác $ABC$ có $A(1;6)$, $B(6;5)$, $C(6;1)$.
a) Tìm tọa độ $M$ sao cho $\overrightarrow {CM} = \overrightarrow {CA} – \overrightarrow {CB} $
b) Đường tròn ngoại tiếp tam giác $ABC$ cắt trục tung tại hai điểm phân biệt $E$, $F$. Tìm tọa độ tâm đường tròn ngoại tiếp tam giác $ABC$. Tìm toạ độ $E$ và $F$.

Lời giải

Bài 1.
a) $\dfrac{{{x^4} – 10{x^2} + 9}}{{\sqrt {x – 2} }} = 0 \quad (1) $
Điều kiện: $x>2$
$(1) \Leftrightarrow {x^4} – 10{x^2} + 9 =0 \Leftrightarrow \left[ \begin{array}{l}
x=1 \quad (l) \\
x=-1 \quad (l) \\
x=3 \quad (n) \\
x=-3 \quad (l)
\end{array} \right. $
Vậy $S=\left\{ 3 \right\} $
b) $x\sqrt{x^2-x+3} = x(x-6)$ (NX: $x^2 -x+3 >0$, $\forall x\in \mathbb{R}$)
$\Leftrightarrow \left[ \begin{array}{l}
x=0\\
\sqrt{x^2 -x +3 } = x-6 \ (*)
\end{array}\right. $
$(*)\Leftrightarrow \left\{ \begin{array}{l} x-6\ge 0\\
x^2 -x +3 = (x-6)^2
\end{array}\right. $
$\Leftrightarrow \left\{ \begin{array}{l}
x\ge 6\\
x=3
\end{array}\right. $
$\Leftrightarrow x\in \emptyset$
Vậy $S=\left\{ 0\right\} $

Bài 2. (1 điểm) ĐKXĐ: $x\ne 0$, $x\ne 1$

Phương trình trở thành: $(m+2)x=1$

Phương trình có nghiệm duy nhất khi và chỉ khi $\left\{ \begin{array}{l}
m+2\ne 0\\\\
\dfrac{1}{m+2}\ne 0\\\\
\dfrac{1}{m+2}\ne 1
\end{array}\right. $
$\Leftrightarrow \left\{ \begin{array}{l}
m\ne -2\\
m\ne -1
\end{array}\right. $

Vậy $m\ne -2$ và $m\ne -1$ thì phương trình có nghiệm duy nhất $x=\dfrac{1}{m+2}$
Bài 3. (1 điểm)
$VT= \left[ {\cos 2\pi – \cos \left( {2\pi + x} \right)} \right]\left[ {1 + {{\tan }^2}\left( {\dfrac{\pi }{2} – x} \right)} \right] $

$= (1-\cos x) (1+\cot^2 x)$

$ = (1-\cos x) \cdot \dfrac{1}{\sin^2 x}$

$= (1-\cos x )\cdot \dfrac{1}{1-\cos^2 x}$

$=\dfrac{1}{1+\cos x}=VP$
Bài 4. (1 điểm)
a) Ta có:
$D=\left| \begin{array}{*{20}{c}}
{m}&{-(m+1)}\\
{2-m}&{m-3}
\end{array}\right| = 2(1-m)$

$D_x = \left| \begin{array}{*{20}{c}}
{1} & {-(m+1)}\\
{3-2m} & {m-3}
\end{array}\right| = 2m(1-m)$

$D_y=\left| \begin{array}{*{20}{c}}
{m} & {1}\\
{2-m} & {3-2m}
\end{array}\right| = -2(m-1)^2$

Hệ phương trình có nghiệm duy nhất khi và chỉ khi $D\ne 0 \Leftrightarrow m\ne 1$
b) Ta có: $\left\{ \begin{array}{l}
x_0 = \dfrac{D_x}{D} = m\\\\
y_0= \dfrac{D_y}{D} = m-1
\end{array}\right. $

Ta có: $x_0^2 – y_0^2 -2x_0 = m^2 – (m-1)^2 -2m =-1$
Bài 5. (1 điểm) Thay $M(0;4)$ vào $(P)$, ta có: $4=-m \Leftrightarrow m=-4$
Tọa độ đỉnh $I( -1;3)$
Bài 6. (2 điểm)
a) Ta có: $\overrightarrow{DA} \cdot \overrightarrow{AB} = -\overrightarrow{AD} \cdot \overrightarrow{AB} = – AD \cdot AB \cdot \cos 120^\circ = a^2$

Ta có: $AB^2 – AD^2 = \left( \overrightarrow{AB}\right) ^2 – \left( \overrightarrow{ AD}\right) ^2 $

$= \left( \overrightarrow{AB} – \overrightarrow{AD}\right) \left( \overrightarrow{AB} + \overrightarrow{AD} \right) = \overrightarrow{DB} \cdot \overrightarrow{AC}$
b) Đặt $\overrightarrow{DH} =x\overrightarrow{DB}$

Ta có: $\overrightarrow{AH} = x\overrightarrow{AB} + (1-x)\overrightarrow{AD}$

Ta có: $\overrightarrow{AH} \cdot \overrightarrow{BD} = 0$

$\Leftrightarrow \left( x\overrightarrow{AB} + (1-x)\overrightarrow{AD}\right) \cdot \left( \overrightarrow{AD} – \overrightarrow{AB}\right) =0$

$\Leftrightarrow x (-a^2) -4xa^2 + (1-x)a^2 -(1-x)(-a^2) =0$

$\Leftrightarrow x=\dfrac{2}{7}$

Ta có: $\overrightarrow{DH} = \dfrac{2}{7} \overrightarrow{DB}$

$\Rightarrow \overrightarrow{DA} \cdot \overrightarrow{DH} = \dfrac{2}{7} \overrightarrow{DA} \cdot \overrightarrow{DB}$

$=\dfrac{2}{7} \overrightarrow{DA} \left( \overrightarrow{DA} + \overrightarrow{AB}\right) $

$=\dfrac{2}{7} \left( DA^2 + \overrightarrow{DA} \cdot \overrightarrow{AB}\right) $

$=\dfrac{4}{7}a^2$
Bài 7. (2 điểm)
a) Gọi $M(x;y)$

Ta có: $\overrightarrow{CM} = \overrightarrow{CA} – \overrightarrow {CB}$
$\Leftrightarrow \overrightarrow{CM} = \overrightarrow{BA}$
$\Leftrightarrow \left\{ \begin{array}{l}
x-6 = -5\\
y-1=1
\end{array}\right. $
$\Leftrightarrow \left\{ \begin{array}{l}
x=1\\
y=2
\end{array}\right. $

Vậy $M(1;2)$
b) Gọi $I(x_I;y_I)$ là tâm đường tròn ngoại tiếp tam giác $ABC$.

Ta có: $\left\{ \begin{array}{l}
IA = IB\\
IA = IC
\end{array}\right. $
$\Rightarrow \left\{ \begin{array}{l}
5x_I -y_I =12\\
(5-y_I)^2 = (1-y_I)^2
\end{array}\right. $
$\Rightarrow \left\{ \begin{array}{l}
x_I=3\\
y_I=3
\end{array}\right. $

Gọi $E(0;y_E)\in Oy$.

Ta có: $IA = IE \Rightarrow (3-y_E)^2 =4 \Rightarrow \left[ \begin{array}{l}
y_E =1\\
y_E =5
\end{array}\right. $

Vậy $E(0;1)$, $F(0;5)$ hoặc ngược lại.

Đáp án đề thi chọn đội tuyển trường PTNK năm 2020

Ngày thi thứ nhất.

Bài 1. Với mỗi số nguyên dương $n$, tìm số thực $M_{n}$ lớn nhất sao cho với mọi số thực dương $x_{1}, x_{2}, \ldots, x_{n}$ thì ta đều có
$$
\sum_{k=1}^{n} \frac{1}{x_{k}^{2}}+\frac{1}{\left(\sum_{k=1}^{n} x_{k}\right)^{2}} \geq M_{n}\left(\sum_{k=1}^{n} \frac{1}{x_{k}}+\frac{1}{\sum_{k=1}^{n} x_{k}}\right)^{2}
$$

Bài 2. Cho 2021 số nguyên khác 0 . Biết rằng tổng của một số bất kỳ trong chúng với tích của tất cả 2020 số còn lại luôn âm.
(a) Chứng minh rằng với mọi cách chia 2021 số này thành hai nhóm và nhân các số cùng nhóm lại với nhau thì tổng của hai tích cũng luôn âm.
(b) Một bộ số thỏa mãn đề bài thì có thể có nhiều nhất mấy số âm?

Bài 3. Cho hai hàm số $f: \mathbb{R} \rightarrow \mathbb{R}$ và $g: \mathbb{R} \rightarrow \mathbb{R}$ thỏa mãn $g(2020)>0$ và với mọi $x, y \in \mathbb{R}$ thì $\left\{\begin{array}{l}f(x-g(y))=f(-x+2 g(y))+x g(y)-6 \\ g(y)=g(2 f(x)-y)\end{array}\right.$

(a) Chứng minh rằng $g$ là hàm hằng.

(b) Chứng minh rằng đồ thị $h(x)=f(x)-x$ nhận $x=1$ là trục đối xứng.

Bài 4. Cho tam giác $A B C$ nhọn, nội tiếp trong đường tròn $(O)$ có trực tâm $H$ và $A H, B H, C H$ cắt cạnh đối diện lần lượt tại $D, E, F$. Gọi $I, M, N$ lần lượt là trung điểm các cạnh $B C, H B, H C$ và $B H, C H$ cắt lại $(O)$ theo thứ tự tại các diểm $L, K$. Giả sử $K L$ cắt $M N$ ở $G$.
(a) Trên $E F$, lấy điểm $T$ sao cho $A T$ vuông góc với $H I$. Chứng minh rằng $G T$ vuông góc với $O H$.
(b) Gọi $P, Q$ lần lượt là giao điểm của $D E, D F$ và $M N$. Gọi $S$ là giao điểm của $B Q, C P$. Chứng minh rằng $H S$ di qua trung điểm của $E F$.

Ngày thi thứ hai.
Bài 5. Cho số nguyên dương $n>1$. Chứng minh rằng với mọi số thực $a \in\left(0 ; \frac{1}{n}\right)$ và mọi đa thức $P(x)$ có bậc $2 n-1$ thỏa mãn điều kiện $P(0)=P(1)=0$, luôn tồn tại các số thực $x_{1}, x_{2}$ thuộc $[0 ; 1]$ sao cho $P\left(x_{1}\right)=P\left(x_{2}\right)$ và $x_{2}-x_{1}=a$.

Bài 6. Giải phương trình sau trên $\mathbb{Z}^{+}:\left(x^{2}+3\right)^{3^{x+1}}\left[\left(x^{2}+3\right)^{3^{x+1}}+1\right]+x^{2}+y=x^{2} y$.

Bài 7 . Cho các số nguyên $n>k>t>0$ và $X={1,2, \ldots, n}$. Gọi $\mathcal{F}$ là họ các tập con có $k$ phần tử của tập hợp $X$ sao cho với mọi $F, F^{\prime} \in \mathcal{F}$ thì $\left|F \cap F^{\prime}\right| \geq t$. Giả sử không có tập con có $t$ phần tử nào chứa trong tất cả các tập $F \in \mathcal{F}$.
(a) Chứng minh rằng tồn tại một tập hợp $B \subset X$ sao cho $|B|<3 k$ và $|B \cap F| \geq t+1$ với mọi $F \in \mathcal{F}$.
(b) Chứng minh rằng $|\mathcal{F}|<C_{3 k}^{t+1} C_{n}^{k-t-1}$.

Bài 8. Cho tam giác $A B C$ nội tiếp trong $(O)$ với $B, C$ cố định và $A$ thay đổi trên cung lớn $B C$. Dựng hình bình hành $A B D C$ và $A D$ cắt lại $(B C D)$ ở $K$.
(a) Gọi $R_{1}, R_{2}$ lần lượt là bán kính đường tròn ngoại tiếp $(K A B),(K A C)$. Chứng minh rằng tích $R_{1} R_{2}$ không đổi.
(b) Ký hiệu $(T),\left(T^{\prime}\right)$ lần lượt là các đường tròn cùng đi qua $K$, tiếp xúc với $B D$ ở $B$ và tiếp xúc với $C D$ ở $C$. Giả sử $(T),\left(T^{\prime}\right)$ cắt nhau ở $L \neq K$. Chứng minh rằng $A L$ luôn đi qua một điểm cố định.

Hết

Đề thi vào lớp 10 Chuyên Toán vào trường PTNK năm 2020

ĐỀ BÀI

Bài 1.  Cho các phương trình: $x^2+ ax +3=0$ và $x^2 +bx +5=0$ với $a$, $b$ là tham số. a) Chứng minh nếu $ab\ge 16$ thì trong hai phương trình trên có ít nhất một phương trình có nghiệm. b) Giả sử hai phương trình trên có nghiệm chung $x_0$. Tìm $a$, $b$ sao cho $|a|+|b|$ có giá trị nhỏ nhất. Bài 2. Cho phương trình: $3x^2-y^2=23^n$ với $n$ là số tự nhiên. a) Chứng minh nếu $n$ chẵn thì phương trình đã cho không có nghiệm nguyên $(x,y)$. b) Chứng minh nếu $n$ lẻ thì phương trình đã cho có nghiệm nguyên $(x,y)$. Bài 3.  Cho đường tròn $(O)$, dây cung $BC$ không chứa tâm $O$ và điểm $A$ thay đổi trên cung lớn $BC$. Lấy các điểm $E$ và $F$ thỏa mãn: $\angle ABE =\angle CAE =\angle ACF =\angle BAF =90^\circ $. a) Chứng minh rằng $AE\cdot AC =AF \cdot AB$ và điểm $O$ là trung điểm $EF$. b) Hạ $AD$ vuông góc với $EF$ $(D\in EF)$. Chứng minh các tam giác $DAB$ và $DCA$ đồng dạng và điểm $D$ thuộc một đường tròn cố định. c) Gọi $G$ là giao điểm của $AD$ với đường tròn $(O)$ $(G\ne A)$. Chứng minh $AD$ đi qua một điểm cố định và $GB\cdot AC = GC\cdot AB$. d) Gọi $K$ là tâm đường tròn ngoại tiếp tam giác $AEF$. Chứng minh $AK$ đi qua một điểm cố định. Bài 4.  Cho số tự nhiên $a=3^{13}\cdot 5^7 \cdot 7^{20}$ a) Gọi $A$ là tập hợp các số nguyên dương $k$ sao cho $k$ là ước của $a$ và $k$ chia hết cho 105. Hỏi tập $A$ có bao nhiêu phần tử? b) Giả sử $B$ là một tập con bất kỳ của $A$ có 9 phần tử. Chứng minh ta luôn có thể tìm được 2 phần tử của $B$ sao cho tích của chúng là số chính phương. Bài 5. Cho hệ phương trình với $k$ là tham số: $\left\{ \begin{array}{l} \dfrac{x}{\sqrt{yz}}+\sqrt{\dfrac{x}{y}}+\sqrt{\dfrac{x}{z}}=k\\ \dfrac{y}{\sqrt{zx}}+\sqrt{\dfrac{y}{z}}+\sqrt{\dfrac{y}{x}}=k\\ \dfrac{z}{\sqrt{xy}}+\sqrt{\dfrac{z}{x}}+\sqrt{\dfrac{z}{y}}=k \end{array} \right. $ a) Giải hệ với $k=1$. b) Chứng minh hệ vô nghiệm với $k\ge 2$ và $k\ne 3$.

LỜI GIẢI

Bài 1.  Xét phương trình: $x^2 +ax +3=0 \quad (1)$, ta có: $\Delta_1 = a^2-12$. Xét phương trình: $x^2 +bx +5=0 \quad (2)$, ta có: $\Delta_2 = b^2-20$ Ta có: $\Delta_1 + \Delta_2 = a^2 + b^2 -32 \ge 2ab -32 \ge 0$ Vậy trong hai số $\Delta_1$ và $\Delta_2$ có ít nhất một số không âm hay một trong hai phương trình đã cho có nghiệm. Dễ thấy $x_0 \ne 0$.
  • $(1) \Leftrightarrow -a=\dfrac{x_0^2+3}{x_0} \Leftrightarrow |a|=\dfrac{x_0^2+3}{|x_0|}$ $(2) \Leftrightarrow -b=\dfrac{x_0^2+5}{x_0} \Leftrightarrow |b|=\dfrac{x_0^2+5}{|x_0|}$
  • Suy ra $|a|+|b|= 2|x_0| + \dfrac{8}{|x_0|} \ge 2\sqrt{2|x_0| \cdot \dfrac{8}{|x_0|}} =8 $
Dấu $”=”$ xảy ra khi và chỉ khi: $x_0^2=4 \Leftrightarrow \left[ \begin{array}{l} x_0 =2 \\ x_0 = -2 \end{array} \right. $ Với $x_0=2$ hoặc $x_0=-2$, lần lượt giải được $a=\dfrac{7}{2}; \, b= \dfrac{9}{2}$ hoặc \ $a=-\dfrac{7}{2}; \, b=- \dfrac{9}{2}$ Vậy giá trị nhỏ nhất của $|a|+|b|$ là 8 khi $a=\dfrac{7}{2}; \, b= \dfrac{9}{2}$ hoặc $a=-\dfrac{7}{2}; \, b=- \dfrac{9}{2}$ Bài 2. a) Ta nhận thấy 1 số chính phương $m=a^2$ khi chia cho 3 thì có số dư lần lượt là 0 hoặc 1. Nên tổng 2 số chính phương nếu chia hết cho 3 thì mỗi số đều phải chia hết cho 3. Quay lại bài toán, do $n$ chẵn nên $23^n$ và $y^2$ đều là các số chính phương mà $23^n +y^2 =3x^2\ \vdots \ 3 \Rightarrow 23^n\ \vdots \ 3$ (vô lí) Vậy $n$ chẵn thì phương trình đã cho không có nghiệm nguyên. b) Do $n$ lẻ $\Rightarrow n=2k+1$ ($k\in \mathbb{N^*}$) Xét $\left\{ \begin{array}{l} x=3\cdot 23^k\\ y=2\cdot 23^k \end{array}\right. $ $\Rightarrow 3x^2-y^2=23^{2k+1}=23^n$ Vậy phương trình có nghiệm nguyên Bài 3.
a) Ta có $\angle BAE + \angle EAF = 90^\circ$ và $\angle CAF + \angle EAF = 90^\circ$. Suy ra $\angle BAE = \angle CAF$. $\triangle ABE \backsim \triangle ACF$, suy ra $AE \cdot AC = AB \cdot AF$ Gọi $I$ là giao điểm của $BE$ và $CF$. Khi đó $AI$ là đường kính của $O$. Tứ giác $AEIF$ là hình bình hành, $O$ là trung điểm $AI$ nên là trung điểm $EF$. b) Các tứ giác $ADBE, ADFC$ nội tiếp. Khi đó $\angle ADB = \angle AEB = \angle AFC = \angle ACD$. $\angle ABD = \angle AEC = \angle IFE = \angle AFC = \angle ADC$. Suy ra $\triangle ADB \backsim \triangle ACDA$. (g.g) Ta có $\angle BDC = 2 \angle ADB = 2 \angle AEB = 2 \angle EIF = \angle BOC$. Suy ra tứ giác $BDOC$ nội tiếp. $D$ thuộc đường tròn ngoại tiếp tam giác $BOC$ cố định. c)  Gọi $S$ là giao điểm của $AD$ và $(BOC$), ta có $\angle OBS = \angle ODS = 90^\circ$. Suy ra $OS$ là đường kính của $(BOC$, do đó $S$ cố định. $AD$ qua $S$ cố định và $SB, SC$ là tiếp tuyến của $(O)$. Khi đó $\triangle SAB \backsim \triangle SGB$, suy ra $\dfrac{AB}{BG} = \dfrac{SB}{SG}$ tương tự thì $\dfrac{AC}{GC} = \dfrac{SC}{SG}$. Mà $SB = SC$, nên $\dfrac{AB}{BG} = \dfrac{AC}{CG}$, suy ra $GB \cdot AC = GC \cdot AB$. Dễ thấy $D$ là trung điểm của $AG$. d) Gọi $M$ là trung điểm của $BC$. Ta chứng minh $A, M, K$ thẳng hàng. Ta chứng minh được $\angle DAE = \angle KAF$ ($\angle 90^\circ – \angle AED$). Gọi $T$ là trung điểm $CG$. Ta có $\triangle ACD \backsim \triangle BCG$ suy ra $\triangle ABC \backsim \triangle DCG$. Từ đó ta có $\triangle ACM \backsim \triangle DCT$. Khi đó $\angle CAM = \angle CDT = \angle ACD = \angle BAD$. Mà $\angle CAM = \angle CAF + \angle FAM$ và $\angle BAD = \angle BAE + \angle EAD$. Suy ra $\angle FAM = \angle EAD = \angle FAK$. Vậy $A, M, K$ thẳng hàng. $AK$ qua trung điểm $M$ của $BC$ cố định. Bài 4.  a) $k\ \vdots \ 105 \Rightarrow k$ chia hết cho 3, 5, 7 $\Rightarrow k=3^n\cdot 5^m \cdot 7^p$ với $m$, $n$, $p$ nguyên dương $\Rightarrow $ có $13\cdot 7\cdot 20 =1820$ cách. b) Giả sử $B$ là tập hợp 9 số nguyên dương $a_i$, $i=\overline{1,9}$\ với $a_i=3^{n_i}\cdot 5^{m_i}\cdot7^{p_i}$ trong đó $0\le n_i\le 13$; $0\le m_i\le 7$ và $0\le p_i\le 20$ Do $B$ có 9 phân tử. Xét nguyên lý Dirichlet với tập các số $n_i$ thì ta có ít nhất 5 số hạng $a_i$ sao cho các số mũ $n_i$ của 3 tương ứng cùng tính chẵn lẻ. Xét tiếp nguyên lý Dirichlet 5 số này cho số mũ $m_i$ của 5 tương ứng thì ta có ít nhất 3 số mà số mũ $m_i$ cũng cùng tính chẵn lẻ. Với 3 số còn lại này ta cũng xét nguyên lý Dirichlet cho số mũ $p_i$ của 7 thì ta sẽ có ít nhất 2 số cũng tính chẵn lẻ. Do 2 số được chọn này có số mũ cùng tính chẵn lẻ với cả các số 3, 5 và 7 nên tích chúng lại sẽ là số chính phương. Bài 5.  Điều kiện $x, y, z > 0$ hoặc $x, y, z < 0$. Từ hệ ta có $x + \sqrt{xz} + \sqrt{xy} = k\sqrt{yz} (1), y + \sqrt{yz} + \sqrt{yz} = k\sqrt{xz} (2), z +\sqrt{zx}+\sqrt{zy} = k\sqrt{xy} (3)$. a) Khi $k = 1$ ta có $x + \sqrt{xz} + \sqrt{xy} = \sqrt{yz} (1), y + \sqrt{yz} + \sqrt{yz} = \sqrt{xz} (2), z +\sqrt{zx}+\sqrt{zy} = \sqrt{xy} (3)$.
  • Nếu $x, y, z > 0$ thì cộng (3) phương trình ta có vô lí.
  • Nếu $x, y, z < 0$. Cộng 3 phương trình ta có $x+y+z +\sqrt{xy}+\sqrt{xz}+\sqrt{zy} = 0 \Leftrightarrow (\sqrt{-x}-\sqrt{y})^2 +(\sqrt{-y}-\sqrt{-z})^2+(\sqrt{-x}-\sqrt{-z})^2 = 0$, do đó $x=y=z$.
  • Thử lại thấy bộ $(x,y,z)$ mà $x=y=z <0$ thỏa hệ phương trình.
b) Giả sử $k\geq 2, k = 3$ thì hệ có nghiệm $(x,y,z)$. Từ hệ ta có $x+y+z = (k-2)(\sqrt{xy}+\sqrt{xz}+\sqrt{yz}) \geq 0, suy ra $x, y, z > 0$. Giả sử $x = \max{x,y,z}$, ta có $k = \dfrac{x+\sqrt{xy}+\sqrt{xz}}{\sqrt{yz}} \geq 3$. $k = \dfrac{z+\sqrt{xz}+\sqrt{yz}}{\sqrt{xy}} \leq 3$. Do đó $k = 3$ (vô lí). Vậy hệ vô nghiệm khi $k \geq 2 $ và $k \neq 3$.